Anda di halaman 1dari 92

Login ID:8FA03376/Student Name:ANSHULGARG/Overall Score:12

Student ScoreCard
Score:0

Percentile: NA

Test of Logical Reasoning

Score:0

Percentile: NA

Test of Quantitative Aptitude

Score:4

Percentile: NA

Test of VA & RC

Score:8

Percentile: NA

Overall: Overall

Score:12

Percentile: NA

ite
d

Test of Abstract Reasoning

2)

[2]

3)

[3]

4)

[4]

Pr

[1]

es

1)

iv
at
e

Li
m

Question: 1
Find the figure that doesn't fit in the group.

rc

Explanation:
All figure are syummetrical only along the vertical axis. Fig. [4] is the only fig which is symmetric about both the axes. Hence, [4].

[2]

3)

[3]

4)

[4]

ni

2)

ar

[1]

Le

1)

ng

es

ou

Question: 2
Find the figure that doesn't fit in the group.

IM

Explanation:
Fig [3] is the only frame where we have three arrows pointing up and two arrows pointing down. Hence, [3].
Question: 3
Find the figure that doesn't fit in the group.

IMS Learning Resources Pvt.Ltd.,Mumbai.All copyrights to this material vestswith IMS Learning Resources Pvt.Ltd.
No part of this materials either in part oras a whole shall be copied,printed,electronically reproduced,sold or distributed without the written
consent of IMS Learing Resources Pvt.Ltd.and any such violation would entail initiation of suitable legal proceedings.

Copyright

1)

[1]

2)

[2]

3)

[3]

4)

[4]

ite
d

Login ID:8FA03376/Student Name:ANSHULGARG/Overall Score:12

Li
m

Explanation:
Number of arrows is the difference between the number of sides of the polygon and the lines used to write the letter inside the polygon.
This logic is not followed by fig. [3]. Hence, [3].

[2]

3)

[3]

4)

[4]

rc

2)

ou

[1]

es

1)

es

Pr

iv
at
e

Question: 4
Find the figure that doesn't fit in the group.

1)

[1]

2)

Le

ar

ni

ng

Question: 5
Find the figure that doesn't fit in the group.

Explanation:
Only two types of elements are used apart from the digit in each frame except [2]. Hence, [2].

IM

[2]

3)

[3]

4)

[4]

Explanation:
In each option, there are as many number of circles as the number of lines except in option [4]. Hence, [4].
Question: 6
Choose the option which will replace the missing figure in the series represented by the question mark (?).

IMS Learning Resources Pvt.Ltd.,Mumbai.All copyrights to this material vestswith IMS Learning Resources Pvt.Ltd.
No part of this materials either in part oras a whole shall be copied,printed,electronically reproduced,sold or distributed without the written
consent of IMS Learing Resources Pvt.Ltd.and any such violation would entail initiation of suitable legal proceedings.

Copyright

[1]

2)

[2]

3)

[3]

4)

[4]

Li
m

1)

ite
d

Login ID:8FA03376/Student Name:ANSHULGARG/Overall Score:12

iv
at
e

Explanation:
From 1st to 2nd figure columns move one step towards left and from 2nd to 3rd each row moves one step upward and this movement of
rows and columns in alternate steps continues. Hence, [4].

[2]

3)

[3]

4)

[4]

ng

2)

ni

[1]

ar

1)

es

ou

rc

es

Pr

Question: 7
Choose the option which will replace the missing figure in the series represented by the question mark (?).

IM

Le

Explanation:

Question: 8

IMS Learning Resources Pvt.Ltd.,Mumbai.All copyrights to this material vestswith IMS Learning Resources Pvt.Ltd.
No part of this materials either in part oras a whole shall be copied,printed,electronically reproduced,sold or distributed without the written
consent of IMS Learing Resources Pvt.Ltd.and any such violation would entail initiation of suitable legal proceedings.

Copyright

Login ID:8FA03376/Student Name:ANSHULGARG/Overall Score:12

2)

[2]

3)

[3]

4)

[4]

Li
m

[1]

iv
at
e

1)

ite
d

Choose the option which will replace the missing figure in the series represented by the question mark (?).

es

Pr

Explanation:
The arrows below the line moves one step ahead and join the arrows above the line. The arrow head is replaced by it's water image after
moving from below the line to above. The arrows above the line move a step ahead and arrow at the third position disappears and is
replaced by new one below the line. The dissappeared arrow reappears in third box. Therefore, arrow disappeared in second box will
reappear below the line in answer option. Hence, [2].

[2]

3)

[3]

4)

[4]

ar

2)

Le

[1]

1)

ni

ng

es

ou

rc

Question: 9
Choose the option which will replace the missing figure in the series represented by the question mark (?).

IM

Explanation:
The arrow moves clockwise, rotating itself in anti-clockwise direction. The shaded part moves one step in clockwise direction. Hence, [1].
Question: 10
Choose the option which will replace the missing figure in the series represented by the question mark (?).

IMS Learning Resources Pvt.Ltd.,Mumbai.All copyrights to this material vestswith IMS Learning Resources Pvt.Ltd.
No part of this materials either in part oras a whole shall be copied,printed,electronically reproduced,sold or distributed without the written
consent of IMS Learing Resources Pvt.Ltd.and any such violation would entail initiation of suitable legal proceedings.

Copyright

2)

[2]

3)

[3]

4)

[4]

Li
m

[1]

iv
at
e

1)

ite
d

Login ID:8FA03376/Student Name:ANSHULGARG/Overall Score:12

Pr

Explanation:
The three symbols shift in clockwise direction. Hence, [2] is eliminated. Two of the three symbols interchange their positions, while the
third gets replaced. Option [3] is similar to fourth figure and option [4] is similar to second figure (i.e., No elements are replaced.) Hence,
[1].

[2]

3)

[3]

4)

[4]

2)

ng

[1]

ni

1)

es

ou

rc

es

Question: 11
In each of the following questions, in four out of the given five pairs of figures, fig. I is related to fig. II in the same particular manner. Spot
out the pair in which this relationship does not exist between figures I and II.

Le

ar

Explanation:
Only in figure (2), the figure is changed in II rest all have the same figure rotated by 90 clockwise. Hence, [2].

IM

Question: 12
In each of the following questions, in four out of the given five pairs of figures, fig. I is related to fig. II in the same particular manner. Spot
out the pair in which this relationship does not exist between figures I and II.

1)

[1]

2)

[2]

IMS Learning Resources Pvt.Ltd.,Mumbai.All copyrights to this material vestswith IMS Learning Resources Pvt.Ltd.
No part of this materials either in part oras a whole shall be copied,printed,electronically reproduced,sold or distributed without the written
consent of IMS Learing Resources Pvt.Ltd.and any such violation would entail initiation of suitable legal proceedings.

Copyright

Login ID:8FA03376/Student Name:ANSHULGARG/Overall Score:12

3)

[3]

4)

[4]

Explanation:
In all options half of each figure is removed except in [4]. Hence, [4].

2)

[2]

3)

[3]

4)

[4]

es

Explanation:
Figure II in all options is the mirror image of figure I except in [1]. Hence, [1].

iv
at
e

[1]

Pr

1)

Li
m

ite
d

Question: 13
In each of the following questions, in four out of the given five pairs of figures, fig. I is related to fig. II in the same particular manner. Spot
out the pair in which this relationship does not exist between figures I and II.

[2]

3)

[3]

4)

[4]

ng

2)

ni

[1]

ar

1)

es

ou

rc

Question: 14
In each of the following questions, in four out of the given five pairs of figures, fig. I is related to fig. II in the same particular manner. Spot
out the pair in which this relationship does not exist between figures I and II.

Le

Explanation:
Only 4 triangles in figure (II) are filled in all figures except in [4] where 5 triangles are filled. Hence, [4].

IM

Question: 15
In each of the following questions, in four out of the given five pairs of figures, fig. I is related to fig. II in the same particular manner. Spot
out the pair in which this relationship does not exist between figures I and II.

1)

[1]

IMS Learning Resources Pvt.Ltd.,Mumbai.All copyrights to this material vestswith IMS Learning Resources Pvt.Ltd.
No part of this materials either in part oras a whole shall be copied,printed,electronically reproduced,sold or distributed without the written
consent of IMS Learing Resources Pvt.Ltd.and any such violation would entail initiation of suitable legal proceedings.

Copyright

Login ID:8FA03376/Student Name:ANSHULGARG/Overall Score:12

2)

[2]

3)

[3]

4)

[4]

Explanation:
In all the options two elements from second row of I are present in first row of II, except in (1) where are elements of second row of I are
present in first row of II. Hence, [1].

2)

[2]

3)

[3]

4)

[4]

es

[1]

rc

1)

Pr

iv
at
e

Li
m

ite
d

Question: 16
Choose the option that best continues the series.

IM

Le

ar

ni

ng

Question: 17
Choose the option that best continues the series.

es

ou

Explanation:
The centre element takes mirror image and then rotates 90 anticlockwise and clockwise in alternate steps. Bottom right element takes
water image and then rotates 45 clockwise in one step and takes only mirror image in the successive step and so on. Top left element
takes mirror image and then rotates clockwise by 45 and takes water image in alternate steps. Hence, [4].

1)

[1]

2)

[2]

3)

[3]

4)

[4]

Explanation:

IMS Learning Resources Pvt.Ltd.,Mumbai.All copyrights to this material vestswith IMS Learning Resources Pvt.Ltd.
No part of this materials either in part oras a whole shall be copied,printed,electronically reproduced,sold or distributed without the written
consent of IMS Learing Resources Pvt.Ltd.and any such violation would entail initiation of suitable legal proceedings.

Copyright

Login ID:8FA03376/Student Name:ANSHULGARG/Overall Score:12

The numbers increase by '1' from left to right in each step and rotate in anticlockwise direction by 45. The next box will contain 10 which
is rotated through 225 anti-clockwise. Hence, [1].

[1]

2)

[2]

3)

[3]

4)

[4]

Pr

1)

iv
at
e

Li
m

ite
d

Question: 18
Choose the option that best continues the series.

es

[1]

IM

1)

Le

ar

ni

ng

Question: 19
Choose the option that best continues the series.

ou

rc

es

Explanation:

2)

[2]

3)

[3]

4)

[4]

Explanation:

IMS Learning Resources Pvt.Ltd.,Mumbai.All copyrights to this material vestswith IMS Learning Resources Pvt.Ltd.
No part of this materials either in part oras a whole shall be copied,printed,electronically reproduced,sold or distributed without the written
consent of IMS Learing Resources Pvt.Ltd.and any such violation would entail initiation of suitable legal proceedings.

Copyright

Login ID:8FA03376/Student Name:ANSHULGARG/Overall Score:12

[1]

2)

[2]

3)

[3]

4)

[4]

Pr

1)

iv
at
e

Li
m

ite
d

Question: 20
Choose the option that best continues the series.

rc

es

Explanation:
A new element is added at the bottom left corner at each step. And all the elements follow the path of first element. Hence, [3].

IM

Le

ar

ni

ng

es

ou

Question: 21
Refer to the data given below and answer the questions that follow.

Aryan just passed his engineering with 64% aggregate and cleared the aptitude test of Infomax.
1)

[1]

2)

[2]

3)

[3]

4)

[4]

Explanation:

IMS Learning Resources Pvt.Ltd.,Mumbai.All copyrights to this material vestswith IMS Learning Resources Pvt.Ltd.
No part of this materials either in part oras a whole shall be copied,printed,electronically reproduced,sold or distributed without the written
consent of IMS Learing Resources Pvt.Ltd.and any such violation would entail initiation of suitable legal proceedings.

Copyright

Login ID:8FA03376/Student Name:ANSHULGARG/Overall Score:12

Aryan fulfills the 1stand 2ndconditions but data is not given about 3rdor 4thconditions. Hence, [4].

1)

[1]

2)

[2]

3)

[3]

4)

[4]

ite
d

Question: 22
Refer to the data given below and answer the questions that follow.
Salil is working with 'Hexagon' for the last 2 years. He did not clear the aptitude test but cleared the interview. He has an engineering
aggregate of 70%.

Li
m

Explanation:
Only condition (ii) is not satisfied. He will be sent to Mysore training center. Hence, [2].

2)

[2]

3)

[3]

4)

[4]

Pr

[1]

es

1)

iv
at
e

Question: 23
Refer to the data given below and answer the questions that follow.
Devendra has 1 year experience in the software field. He cleared the aptitude test as well as the interview. He is an engineer with 65%.

rc

Explanation:
Devendra has fulfilled all the four conditions hence he is selected. Hence, [1].

[2]

3)

[3]

4)

[4]

2)

ng

[1]

ni

1)

es

ou

Question: 24
Refer to the data given below and answer the questions that follow.
Monish has completed his engineering 2 years back with an aggregate of 65%. He has cleared the aptitude test and the
personal/technical interview. He has no work experience.

Le

ar

Explanation:
Monish has cleared the aptitude test and interview. Only condition (iv) is not satisfied. Hence he will be sent for special training. Hence,
[3].

IM

Question: 25
Refer to the data given below and answer the questions that follow.
Shubham who cleared the aptitude test but not the interview has an aggregate of 69% in engineering graduation. He has 2 years of work
experience.
1)

[1]

2)

[2]

3)

[3]

4)

[4]

Explanation:

IMS Learning Resources Pvt.Ltd.,Mumbai.All copyrights to this material vestswith IMS Learning Resources Pvt.Ltd.
No part of this materials either in part oras a whole shall be copied,printed,electronically reproduced,sold or distributed without the written
consent of IMS Learing Resources Pvt.Ltd.and any such violation would entail initiation of suitable legal proceedings.

Copyright

Login ID:8FA03376/Student Name:ANSHULGARG/Overall Score:12

Shubham has failed to fulfill the 3rdcondition, but he has fulfilled all the remaining conditions. Hence, he will be sent to the Mysore training
centre. Hence, [2].
Question: 26
Each passage is followed by a question. Read the passage carefully and answer the question that follows.
All the songs I have heard from this singer's first album have been very good. I guess the songs on her upcoming album will be good as
well.
Which of the following most closely parallels the reasoning of the preceding argument?
I have eaten Italian food only once, and I liked it very much; I guess Italian cuisine must be very tasty in general.

2)

I think the articles in this newspaper are very well written, which is to be expected, as it employs award-winning journalists.

3)

This artist paints in a very distinctive style. So the painting I just saw must be by him as well, since it was in the same style.

4)

I have read only the latest book by this author, but it was so interesting that it made me keen to try out his earlier book as
well.

Li
m

ite
d

1)

Pr

iv
at
e

Explanation:
The reasoning in the main statement is as follows: because one item produced by someone is good, the other item made by them is also
expected to be good. The same kind of reasoning appears only in [4]. [1] is close, but not quite the same. In the main statement, the
qualities in one instance are expected to be found in another instance as well; in [1], on the other hand, the qualities of one instance are
expected to be found in all such instances in general. [2] is completely wrong, as it merely provides a reason for something being good. [3]
is just about identifying a creator's style, not about how good it is. Hence, [4].

ou

rc

es

Question: 27
Each passage is followed by a question. Read the passage carefully and answer the question that follows.
A spinach packed with HIV suppressing proteins may be the first step in the use of plants as a cheap and safe method of delivering AIDS
vaccines to those who need them. "The ideal situation would be a prescription for a bowl of spinach" that would either help prevent or treat
HIV infection, said lead study author Dr. Alexander Karasev of Thomas Jefferson University in Philadelphia.
Which of the following best undermines the claim made in the passage?
Digestion of any solid food is difficult for many AIDS patients.

2)

When boiled for too long, spinach tends to lose its fortifying proteins.

3)

The variety of spinach that is packed with HIV suppressing proteins is rare.

4)

Most people do not like the taste of spinach.

ng

es

1)

Le

ar

ni

Explanation:
The claim made in the passage is that spinach packed with HIV suppressants will prove beneficial to those who may need AIDS vaccines.
There is no indication in the passage that spinach has to be consumed in its solid form, so option [1] does not undermine the claim. Option
[2] is not a strong reason as the boiling could be regulated. The main claim in the passage is that 'spinach packed with HIV suppressants
would be a safe and cheap method of delivering AIDS vaccines' but if the particular variety of spinach is rare, it may not necessarily be
cheap. Thus, option [3] is the answer. [4] is irrelevant because if something has medicinal properties, its taste is a matter of secondary
consideration. Hence, [3].

IM

Question: 28
Find the odd term in series.
12, 36, 328, 2916, 52488, 1417176
1)

36

2)

328

3)

2916

4)

52488

Explanation:

IMS Learning Resources Pvt.Ltd.,Mumbai.All copyrights to this material vestswith IMS Learning Resources Pvt.Ltd.
No part of this materials either in part oras a whole shall be copied,printed,electronically reproduced,sold or distributed without the written
consent of IMS Learing Resources Pvt.Ltd.and any such violation would entail initiation of suitable legal proceedings.

Copyright

Login ID:8FA03376/Student Name:ANSHULGARG/Overall Score:12

The logic of the series is, a term multiplied with the sum of its digits yields the next term.
Therefore, we can see that 328 neither comes from the given rule nor gives the correct number according to the rule. Thus, 328 is the
wrong number.
Hence, [2].

46

2)

106

3)

426

4)

1706

Li
m

1)

ite
d

Question: 29
Find the odd term in series.
6, 46, 106, 426, 1706, 6826

es

Pr

iv
at
e

Explanation:

3)

43

4)

150

ou

11

es

2)

ng

1)

rc

Question: 30
Find the odd term in series.
3, 11, 43, 150, 683

IM

Le

ar

ni

Explanation:

Question: 31
Find the odd term in series.
GT, KP, CX, EV, QI

IMS Learning Resources Pvt.Ltd.,Mumbai.All copyrights to this material vestswith IMS Learning Resources Pvt.Ltd.
No part of this materials either in part oras a whole shall be copied,printed,electronically reproduced,sold or distributed without the written
consent of IMS Learing Resources Pvt.Ltd.and any such violation would entail initiation of suitable legal proceedings.

Copyright

Login ID:8FA03376/Student Name:ANSHULGARG/Overall Score:12

1)

GT

2)

KP

3)

CX

4)

QI

Li
m

ite
d

Explanation:

M2O

2)

P6S

3)

C120H

4)

X2Y

Pr

1)

iv
at
e

Question: 32
Find the odd term in series.
M2O, P6S, C120H, A24E, X2Y

es

ou

rc

es

Explanation:

Le

ar

ni

ng

Question: 33
Read the short passage given below and answer the question that follows.
State Bank of India (SBI) has swung into action to check its rising operating expenses, one of the key factors responsible for dragging
down its fourth quarter profits. As a first step, it has decided to scale down its branch expansion plan and is now expected to open around
500 branches during the current financial year, as against nearly 1,000 proposed earlier. The country's largest lender, which was on a
branch opening spree, had opened around 1,000 branches and nearly 8,000 automated teller machines (ATMs) in 2009-10. "SBI started
expanding in a big way in 2008-09 and 2009-10. However, the returns have not accrued to the magnitude that was expected."
From the information given above, it can be validly concluded that:
SBI is on a cost-cutting drive, and will go slow on branch expansion.

2)

SBI is planning aggressive branch expansion after a short but much needed cost-cutting drive.

1)

By controlling costs, the bank could use some of the resources to improve the loan-loss coverage ratio, which is provisions
for bad debt.

4)

SBI's expansion plan met with much success as predicted and expected.

IM
3)

Explanation:
From the given information, one can validly conclude [1]. The opening line mentions 'swung into action to check its rising operating
expenses'. The passage offers clues such as, 'decided to scale down its branch expansion plan' and the concluding line which says,
'returns have not accrued to the magnitude that was expected'. Hence, [1].

IMS Learning Resources Pvt.Ltd.,Mumbai.All copyrights to this material vestswith IMS Learning Resources Pvt.Ltd.
No part of this materials either in part oras a whole shall be copied,printed,electronically reproduced,sold or distributed without the written
consent of IMS Learing Resources Pvt.Ltd.and any such violation would entail initiation of suitable legal proceedings.

Copyright

Login ID:8FA03376/Student Name:ANSHULGARG/Overall Score:12

Question: 34
Choose the correct alternative.
Six friends stand for a parade facing the North. 'Q' has more people to his right than to his left. There is exactly one person between 'S'
and 'L'. 'M' and 'R' stand at the extreme positions and 'O' is the only other person in the parade. Which of the following is definitely true?
Q stands to the immediate left of R.

2)

Q stands second to the right of S.

3)

O stands second to the right of Q.

4)

None of these.

ite
d

1)

iv
at
e

Li
m

Explanation:

Question: 35
Choose the correct alternative.
How many pairs of letters in the word 'ACCIDENTAL' have the same number of letters between them as in the English alphabet series?
None

2)

One

3)

Two

4)

Three

es

Pr

1)

ni

ng

es

ou

rc

Explanation:

IM

Le

ar

Question: 36
Read the following information followed by five statements carefully, consider it to be true and answer the questions that follow.

IMS Learning Resources Pvt.Ltd.,Mumbai.All copyrights to this material vestswith IMS Learning Resources Pvt.Ltd.
No part of this materials either in part oras a whole shall be copied,printed,electronically reproduced,sold or distributed without the written
consent of IMS Learing Resources Pvt.Ltd.and any such violation would entail initiation of suitable legal proceedings.

Copyright

Li
m

ite
d

Login ID:8FA03376/Student Name:ANSHULGARG/Overall Score:12

2)

3)

4)

Pr

1)

iv
at
e

Which of the following, if true, most strengthens the finding of the study?

ou

rc

es

Explanation:
The study concluded that men who played contact sports are more prone to unfriendly and violent behaviour as compared to men who
played noncontact sports. This is most supported by option A. B weakens the finding. Options C and D neither support nor weaken the
finding presented in the passage. Hence, [1].

3)

4)

2)

ng

ni

1)

es

Question: 37
Read the following information followed by five statements carefully, consider it to be true and answer the questions that follow.
Which of the following, if true, most weakens the finding of the study?

IM

Le

ar

Explanation:
Option B, which talks about boys having anger management issues being advised to take up team sports (contact sports), most weakens
the argument in the passage that men who played contact sports during their teenage are more prone to unfriendly and violent behaviour.
If playing contact sports increased or encouraged violent behaviour then psychologists would not advise it for young boys who had anger
management issues. It is not necessary that a person who is boisterous is also violent and/or unfriendly, so E does not weaken the
argument given in the passage. Hence, [2].
Question: 38
Refer to the data below and answer the questions that follow.

IMS Learning Resources Pvt.Ltd.,Mumbai.All copyrights to this material vestswith IMS Learning Resources Pvt.Ltd.
No part of this materials either in part oras a whole shall be copied,printed,electronically reproduced,sold or distributed without the written
consent of IMS Learing Resources Pvt.Ltd.and any such violation would entail initiation of suitable legal proceedings.

Copyright

ite
d

Login ID:8FA03376/Student Name:ANSHULGARG/Overall Score:12

1)

2)

3)

4)

None of these

Li
m

Who stands between Q and P?

es

ou

rc

es

Pr

iv
at
e

Explanation:

Hence, [3].

3)

4)

IM

Explanation:

ni

2)

ar

Le

1)

ng

Question: 39
Refer to the data below and answer the questions that follow.
How many students could be standing to the right of R?

IMS Learning Resources Pvt.Ltd.,Mumbai.All copyrights to this material vestswith IMS Learning Resources Pvt.Ltd.
No part of this materials either in part oras a whole shall be copied,printed,electronically reproduced,sold or distributed without the written
consent of IMS Learing Resources Pvt.Ltd.and any such violation would entail initiation of suitable legal proceedings.

Copyright

Li
m

ite
d

Login ID:8FA03376/Student Name:ANSHULGARG/Overall Score:12

Hence, [2].

To the immediate right of D

2)

Second to the left of A

3)

Third to the left of P

4)

All of these

es

Pr

1)

iv
at
e

Question: 40
Refer to the data below and answer the questions that follow.
Where does T stand?

ni

ng

es

ou

rc

Explanation:

ar

Hence, [3].

IM

Le

Question: 41
Arrange the six sentences given below to form a coherent paragraph and then answer the questions that follow.

IMS Learning Resources Pvt.Ltd.,Mumbai.All copyrights to this material vestswith IMS Learning Resources Pvt.Ltd.
No part of this materials either in part oras a whole shall be copied,printed,electronically reproduced,sold or distributed without the written
consent of IMS Learing Resources Pvt.Ltd.and any such violation would entail initiation of suitable legal proceedings.

Copyright

ite
d

Login ID:8FA03376/Student Name:ANSHULGARG/Overall Score:12

2)

3)

4)

iv
at
e

1)

Li
m

Which will be the fifth sentence of the rearranged paragraph?

es

Pr

Explanation:

rc

C is the fifth sentence of the rearranged paragraph. Hence, [4].

3)

4)

es

2)

ng

1)

ou

Question: 42
Arrange the six sentences given below to form a coherent paragraph and then answer the questions that follow.
Which will be the first sentence of the rearranged paragraph?

Le

ar

ni

Explanation:

D is the first sentence of the rearranged paragraph. Hence, [1].

IM

Question: 43
Arrange the six sentences given below to form a coherent paragraph and then answer the questions that follow.
Which will be the sixth sentence of the rearranged paragraph?
1)

2)

3)

4)

Explanation:

IMS Learning Resources Pvt.Ltd.,Mumbai.All copyrights to this material vestswith IMS Learning Resources Pvt.Ltd.
No part of this materials either in part oras a whole shall be copied,printed,electronically reproduced,sold or distributed without the written
consent of IMS Learing Resources Pvt.Ltd.and any such violation would entail initiation of suitable legal proceedings.

Copyright

Login ID:8FA03376/Student Name:ANSHULGARG/Overall Score:12

E is the sixth sentence of the rearranged paragraph. Hence, [4].

2)

3)

4)

Li
m

1)

ite
d

Question: 44
Arrange the six sentences given below to form a coherent paragraph and then answer the questions that follow.
Which will be the third sentence of the rearranged paragraph?

iv
at
e

Explanation:

Pr

A is the third sentence of the rearranged paragraph. Hence, [1].

3)

4)

rc

2)

ou

es

1)

es

Question: 45
Arrange the six sentences given below to form a coherent paragraph and then answer the questions that follow.
Which will be the fourth sentence of the rearranged paragraph?

ng

Explanation:

ar

ni

B is the fourth sentence of the rearranged paragraph. Hence, [2].

IM

Le

Question: 46
Refer to the data below and answer the questions thatfollow.

What will be the fourth element in Step IV for the below input?

IMS Learning Resources Pvt.Ltd.,Mumbai.All copyrights to this material vestswith IMS Learning Resources Pvt.Ltd.
No part of this materials either in part oras a whole shall be copied,printed,electronically reproduced,sold or distributed without the written
consent of IMS Learing Resources Pvt.Ltd.and any such violation would entail initiation of suitable legal proceedings.

Copyright

Login ID:8FA03376/Student Name:ANSHULGARG/Overall Score:12

Input : 38 54 96 75 25 21 16 31
1)

2)

3)

4)

20 60 51 42 4 27 45 19

3)

20 60 51 42 4 45 27 19

4)

20 51 60 42 4 45 27 19

rc

es

2)

20 60 51 42 27 4 45 19

ng

1)

ou

Question: 47
Refer to the data below and answer the questions thatfollow.
For the input in the previous question, what will be the final output?

es

Pr

iv
at
e

Li
m

ite
d

Explanation:

Le

ar

ni

Explanation:

Hence, [3].

IM

Question: 48
Refer to the data below and answer the questions thatfollow.

1)

20 60 81 23 87 5

2)

20 60 42 81 5 87

3)

20 81 60 23 87 5

IMS Learning Resources Pvt.Ltd.,Mumbai.All copyrights to this material vestswith IMS Learning Resources Pvt.Ltd.
No part of this materials either in part oras a whole shall be copied,printed,electronically reproduced,sold or distributed without the written
consent of IMS Learing Resources Pvt.Ltd.and any such violation would entail initiation of suitable legal proceedings.

Copyright

Login ID:8FA03376/Student Name:ANSHULGARG/Overall Score:12

4)

20 60 81 23 5 87

iv
at
e

Li
m

ite
d

Explanation:

2)

3)

4)

rc

ou

1)

es

Pr

Question: 49
Refer to the data below and answer the questions thatfollow.

IM

Le

ar

ni

ng

es

Explanation:

Question: 50
Refer to the data below and answer the questions thatfollow.
For the input in the previous question, what will be the final output?
1)

30 81 1 82 54 66 48

IMS Learning Resources Pvt.Ltd.,Mumbai.All copyrights to this material vestswith IMS Learning Resources Pvt.Ltd.
No part of this materials either in part oras a whole shall be copied,printed,electronically reproduced,sold or distributed without the written
consent of IMS Learing Resources Pvt.Ltd.and any such violation would entail initiation of suitable legal proceedings.

Copyright

Login ID:8FA03376/Student Name:ANSHULGARG/Overall Score:12

2)

30 1 81 82 54 66 48

3)

30 81 1 54 82 66 48

4)

30 1 81 54 82 66 48

ite
d

Explanation:

Li
m

Hence, [2].

es

Pr

iv
at
e

Question: 51
Refer to the data below and answer the questions that follow.

2)

O2Y2N5

3)

OYM225

4)

OYN225

ou

O2Y2M5

es

1)

rc

If the password for the 6 pm - 7 pm batch on 29thFebruary 2008 was PAP348, then which of the following was the password for the 11 am
- 12 noon batch?

IM

Le

ar

ni

ng

Explanation:

Hence, [1].
Question: 52
Refer to the data below and answer the questions that follow.
If the password for the 10 am - 11 am batch on 14thFebruary was RZC720, then which of the following was the password for the 1 pm - 2

IMS Learning Resources Pvt.Ltd.,Mumbai.All copyrights to this material vestswith IMS Learning Resources Pvt.Ltd.
No part of this materials either in part oras a whole shall be copied,printed,electronically reproduced,sold or distributed without the written
consent of IMS Learing Resources Pvt.Ltd.and any such violation would entail initiation of suitable legal proceedings.

Copyright

Login ID:8FA03376/Student Name:ANSHULGARG/Overall Score:12

pm batch?
1)

U0F8L8

2)

TDI966

3)

U0F8L9

4)

T9D6I6

Pr

iv
at
e

Li
m

ite
d

Explanation:

es

Hence, [3].

2)

Q0E6Y3

3)

WQQ681

4)

W6Q8Q1

es

QEY063

1)

ou

rc

Question: 53
Refer to the data below and answer the questions that follow.
If the password for the 3 pm - 4 pm batch on 20thFebruary was R1G8B6, then which of the following was the password for the 8 pm - 9 pm
batch?

IM

Le

ar

ni

ng

Explanation:

>
Hence, [1].
Question: 54

IMS Learning Resources Pvt.Ltd.,Mumbai.All copyrights to this material vestswith IMS Learning Resources Pvt.Ltd.
No part of this materials either in part oras a whole shall be copied,printed,electronically reproduced,sold or distributed without the written
consent of IMS Learing Resources Pvt.Ltd.and any such violation would entail initiation of suitable legal proceedings.

Copyright

Login ID:8FA03376/Student Name:ANSHULGARG/Overall Score:12

Refer to the data below and answer the questions that follow.
If the password for the 7 pm - 8 pm batch on 17thFebruary was X3I8M2, then which of the following was the password for the 4 pm - 5 pm
batch?
1)

U0C2D3

2)

UCD023

3)

V1E4G6

4)

UIE4G6

es

Pr

iv
at
e

Li
m

ite
d

Explanation:

rc

Hence, [2].

2)

Q0E6Y3

3)

WQQ681

4)

W6Q8Q1

QEY063

ng

1)

es

ou

Question: 55
Refer to the data below and answer the questions that follow.
If the password for the 4 pm - 5 pm batch on 2ndFebruary was SIE209, then which of the following was the password for the 8 pm - 9pm
batch?

IM

Le

ar

ni

Explanation:

Hence, [3].

IMS Learning Resources Pvt.Ltd.,Mumbai.All copyrights to this material vestswith IMS Learning Resources Pvt.Ltd.
No part of this materials either in part oras a whole shall be copied,printed,electronically reproduced,sold or distributed without the written
consent of IMS Learing Resources Pvt.Ltd.and any such violation would entail initiation of suitable legal proceedings.

Copyright

Login ID:8FA03376/Student Name:ANSHULGARG/Overall Score:12

iv
at
e

Li
m

ite
d

Question: 56
Carefully read the following information and the five statements that follow. Then, answer the questions.

1)

2)

3)

4)

Pr

Which of the following can be inferred about the disease?

rc

es

Explanation:
The passage clearly says that the disease is a 'deadly' one. Thus, we can infer that pancreatic cancer has poor survival rates. Hence, [2].

3)

4)

es

2)

ng

1)

ou

Question: 57
Carefully read the following information and the five statements that follow. Then, answer the questions.
Which of the following statements would strengthen the argument put forth in the passage?

ar

ni

Explanation:
If scientists found that consuming even a small quantity (just 50 grams) of processed meat raises the likelihood of contracting this disease
by 19 percent, we can say that eating processed meat even in small quantities raises the risk of developing this disease. Hence, [3].

IM

1)

Le

Question: 58
Carefully read the following information and the five statements that follow. Then, answer the questions.
Which of the following would be a new cause of concern regarding the disease?

2)

3)

4)

Explanation:
If pancreatic cancer does not produce symptoms in the early stages, then it is a real cause of concern because it cannot be cured when it
is in a nascent stage. In the latter stages it might be difficult to treat the patient and he could succumb to death. B has already been stated
in the paragraph. So, it is not new information. Hence, [1].

IMS Learning Resources Pvt.Ltd.,Mumbai.All copyrights to this material vestswith IMS Learning Resources Pvt.Ltd.
No part of this materials either in part oras a whole shall be copied,printed,electronically reproduced,sold or distributed without the written
consent of IMS Learing Resources Pvt.Ltd.and any such violation would entail initiation of suitable legal proceedings.

Copyright

Login ID:8FA03376/Student Name:ANSHULGARG/Overall Score:12

Question: 59
Carefully read the following information and the five statements that follow. Then, answer the questions.
Which of the following statements would be a suitable course of action?
A

2)

3)

4)

ite
d

1)

Li
m

Explanation:
The passage talks about some information that has been revealed about one cause of pancreatic cancer. Among the options, the only
possible course of action is D. It makes sense to find out about the other causes of this deadly disease.
Hence, [4].

2)

3)

4)

es

1)

ou

rc

es

Pr

iv
at
e

Question: 60
Read the following information followed by five statements carefully, and answer the questions that follow.
Expressive Language Disorder (ELD), characterized by difficulty in expressing oneself beyond simple sentences and a limited vocabulary,
has become a very common communication disorder among teenagers. The number of cases reported this year is more than double the
number reported ten years ago. Experts consider the advent of internet lingo, which majorly consists of acronyms and emoticons, and the
increasing popularity of social networking sites like Facebook and Twitter as the main causes of this disorder.
A. Teenagers who do not use social networking sites like Facebook and Twitter do not suffer from expressive language disorder.
B. Children would no longer pursue reading and writing as hobbies.
C. Facebook and Twitter were not very popular a decade back.
D. Demand for language courses that teach basic reading and writing skills would increase.
E. 90 percent of the teenagers who suffer from ELD are being treated for learning disabilities such as dyslexia.
Which of the following can be inferred from the passage?

ar

ni

ng

Explanation:
The passage says that experts consider increased use of social networking sites like Facebook and Twitter as the main cause, not the
only cause, of this disorder. So, statement A is negated. No information is given in the passage from which statements B or D can be
inferred. Statement C can be inferred from the latter part of the passage. Hence, [3].

1)

2)

Le

Question: 61
Read the following information followed by five statements carefully, and answer the questions that follow.
Which of the following can be a probable effect of the situation described in the passage?

IM

3)

4)

Explanation:
Statements A and B are not effects. Statement B is too extreme to be a probable effect. Only statement D, which talks about the demand
for language courses that teach basic reading and writing skills, can be a probable effect of the situation described in the passage. Hence,
[4].

IMS Learning Resources Pvt.Ltd.,Mumbai.All copyrights to this material vestswith IMS Learning Resources Pvt.Ltd.
No part of this materials either in part oras a whole shall be copied,printed,electronically reproduced,sold or distributed without the written
consent of IMS Learing Resources Pvt.Ltd.and any such violation would entail initiation of suitable legal proceedings.

Copyright

Login ID:8FA03376/Student Name:ANSHULGARG/Overall Score:12

Question: 62
Read the following information followed by five statements carefully, and answer the questions that follow.
Which of the following, if true, would undermine the belief of the experts?
1)

2)

3)

4)

Li
m

ite
d

Explanation:
Statement A strengthens the belief of the experts. Statement E, which links Expressive Language Disorder (ELD) with learning
disabilities, undermines the belief of the experts who consider internet lingo and social networking sites like Facebook and Twitter as the
main causes of this disorder. B and C are irrelevant. Hence, [4].

2)

III and IV follow.

3)

Only III follows.

4)

I and II follow.

rc

Only II follows.

ou

1)

es

Pr

iv
at
e

Question: 63
Each question consists of two premises followed by four conclusions. Choose which of the conclusion/s logically follow/s from both the
premises.
A. Some dwarfs are pixies.
B. No pixie is an elf.
Conclusion I: Some elves may be dwarfs.
Conclusion II: Some dwarfs are not elves.
Conclusion III: Some pixies are dwarfs.
Conclusion IV: No dwarf is an elf.

Le

ar

ni

ng

es

Explanation:

IM

Question: 64
Each question consists of two premises followed by four conclusions. Choose which of the conclusion/s logically follow/s from both the
premises.
A. All novels are books.
B. All books are interesting.
Conclusion I: Some books are novels.
Conclusion II: No novels are interesting.
Conclusion III:Some books may not be novels.

IMS Learning Resources Pvt.Ltd.,Mumbai.All copyrights to this material vestswith IMS Learning Resources Pvt.Ltd.
No part of this materials either in part oras a whole shall be copied,printed,electronically reproduced,sold or distributed without the written
consent of IMS Learing Resources Pvt.Ltd.and any such violation would entail initiation of suitable legal proceedings.

Copyright

Login ID:8FA03376/Student Name:ANSHULGARG/Overall Score:12

Conclusion IV: All novels are interesting.


1)

I and IV follow.

2)

I, II and IV follow.

3)

Only IV follows.

4)

Only II follows.

iv
at
e

Li
m

ite
d

Explanation:

Only IV follows.

3)

II and III follow.

4)

Only III follows.

IM

Le

ar

ni

Explanation:

es

2)

II and IV follow.

ng

1)

ou

rc

es

Pr

Question: 65
Each question consists of two premises followed by four conclusions. Choose which of the conclusion/s logically follow/s from both the
premises.
A. No denims are trousers.
B. Some khakis are trousers.
Conclusion I: Some khakis are denims.
Conclusion II: Some denims are not trousers.
Conclusion III: Some khakis may be denims.
Conclusion IV: No denims are khakis.

Question: 66
Refer to the data below and answer the questions that follow.

IMS Learning Resources Pvt.Ltd.,Mumbai.All copyrights to this material vestswith IMS Learning Resources Pvt.Ltd.
No part of this materials either in part oras a whole shall be copied,printed,electronically reproduced,sold or distributed without the written
consent of IMS Learing Resources Pvt.Ltd.and any such violation would entail initiation of suitable legal proceedings.

Copyright

Li
m

ite
d

Login ID:8FA03376/Student Name:ANSHULGARG/Overall Score:12

1)

22

2)

25

3)

23

4)

30

Pr

Explanation:
Students participating in at least two competitions = 8 + 7 + 4 + 6 + 5 = 30. Hence, [4].

iv
at
e

How many students participated in at least two competitions?

15

3)

11

4)

22

ou

2)

es

18

1)

rc

es

Question: 67
Refer to the data below and answer the questions that follow.
What is difference between number of students participating in dance and essay competitions?

ar

ni

ng

Explanation:

Le

Question: 68
Refer to the data below and answer the questions that follow.
How many students participated in one-act play competition but not in singing or essay competition?
18

2)

1)

3)

26

4)

19

IM

29

Explanation:
Required number of students = 12 + 7 = 19. Hence, [4].
Question: 69

IMS Learning Resources Pvt.Ltd.,Mumbai.All copyrights to this material vestswith IMS Learning Resources Pvt.Ltd.
No part of this materials either in part oras a whole shall be copied,printed,electronically reproduced,sold or distributed without the written
consent of IMS Learing Resources Pvt.Ltd.and any such violation would entail initiation of suitable legal proceedings.

Copyright

Login ID:8FA03376/Student Name:ANSHULGARG/Overall Score:12

Refer to the data below and answer the questions that follow.
How many students participated in more than three of the competitions?
1)

2)

3)

4)

ite
d

Explanation:
No student participated in more than three of the competitions. Hence, [3].

30

2)

34

3)

29

4)

36

iv
at
e

1)

Li
m

Question: 70
Refer to the data below and answer the questions that follow.
Number of students who participated in exactly one of the competitions is how much more than the number of students who participated in
exactly two of the competitions?

es

ou

rc

es

Pr

Explanation:

IM

Le

ar

ni

ng

Question: 71
Refer to the data below and answer the questions that follow.
.

If the person who rides Yamaha is not Vimla's husband then who rides TVS?
1)

Ram

2)

Ravi

IMS Learning Resources Pvt.Ltd.,Mumbai.All copyrights to this material vestswith IMS Learning Resources Pvt.Ltd.
No part of this materials either in part oras a whole shall be copied,printed,electronically reproduced,sold or distributed without the written
consent of IMS Learing Resources Pvt.Ltd.and any such violation would entail initiation of suitable legal proceedings.

Copyright

Login ID:8FA03376/Student Name:ANSHULGARG/Overall Score:12

3)

Vaibhav

4)

Cannot be determined

iv
at
e

Li
m

ite
d

Explanation:

Hence, [4].

ni

ng

es

ou

rc

es

Pr

Question: 72
Refer to the data below and answer the questions that follow.

What does Ram ride if Ravi rides the TVS?


Pulsar

2)

Yamaha

3)

Honda

4)

Cannot be determined

Le

IM

Explanation:

ar

1)

IMS Learning Resources Pvt.Ltd.,Mumbai.All copyrights to this material vestswith IMS Learning Resources Pvt.Ltd.
No part of this materials either in part oras a whole shall be copied,printed,electronically reproduced,sold or distributed without the written
consent of IMS Learing Resources Pvt.Ltd.and any such violation would entail initiation of suitable legal proceedings.

Copyright

Li
m

ite
d

Login ID:8FA03376/Student Name:ANSHULGARG/Overall Score:12

Hence, [2].

es

ou

rc

es

Pr

iv
at
e

Question: 73
Refer to the data below and answer the questions that follow.

Kamla

2)

Sarita

3)

Vimla

4)

Vidya

ar

IM

Le

Explanation:

ni

1)

ng

Who is Saurabh's wife?

IMS Learning Resources Pvt.Ltd.,Mumbai.All copyrights to this material vestswith IMS Learning Resources Pvt.Ltd.
No part of this materials either in part oras a whole shall be copied,printed,electronically reproduced,sold or distributed without the written
consent of IMS Learing Resources Pvt.Ltd.and any such violation would entail initiation of suitable legal proceedings.

Copyright

Li
m

ite
d

Login ID:8FA03376/Student Name:ANSHULGARG/Overall Score:12

Hence, [2].

es

ou

rc

es

Pr

iv
at
e

Question: 74
Refer to the data below and answer the questions that follow.

Who has done a B.A.?


Shon

2)

Vaibhav

3)

Saurabh

4)

None of these

ar

ni

ng

1)

IM

Le

Explanation:

IMS Learning Resources Pvt.Ltd.,Mumbai.All copyrights to this material vestswith IMS Learning Resources Pvt.Ltd.
No part of this materials either in part oras a whole shall be copied,printed,electronically reproduced,sold or distributed without the written
consent of IMS Learing Resources Pvt.Ltd.and any such violation would entail initiation of suitable legal proceedings.

Copyright

Li
m

ite
d

Login ID:8FA03376/Student Name:ANSHULGARG/Overall Score:12

Hence, [1].

es

ou

rc

es

Pr

iv
at
e

Question: 75
Refer to the data below and answer the questions that follow.

Which of the below statements is definitely false?


Vimla's husband is an M.A.

2)

Reena is Ravi's wife.

3)

Reena's husband is a B.A.

4)

Ravi owns a Yamaha.

ar

ni

ng

1)

IM

Le

Explanation:

IMS Learning Resources Pvt.Ltd.,Mumbai.All copyrights to this material vestswith IMS Learning Resources Pvt.Ltd.
No part of this materials either in part oras a whole shall be copied,printed,electronically reproduced,sold or distributed without the written
consent of IMS Learing Resources Pvt.Ltd.and any such violation would entail initiation of suitable legal proceedings.

Copyright

Li
m

ite
d

Login ID:8FA03376/Student Name:ANSHULGARG/Overall Score:12

Hence, [1].

rc

es

Pr

iv
at
e

Question: 76
Refer to the data given below and answer the questions.

If Rakesh is equidistant from both Keshav and Madhav then whose sibling is Sudhir?
Mihir

2)

Sameer

3)

Madhav

4)

None of these

es

ou

1)

IM

Le

ar

ni

ng

Explanation:

IMS Learning Resources Pvt.Ltd.,Mumbai.All copyrights to this material vestswith IMS Learning Resources Pvt.Ltd.
No part of this materials either in part oras a whole shall be copied,printed,electronically reproduced,sold or distributed without the written
consent of IMS Learing Resources Pvt.Ltd.and any such violation would entail initiation of suitable legal proceedings.

Copyright

es

From fig (1) Sudhir is Sameer's sibling. Hence, [2].

ou

rc

es

Pr

iv
at
e

Li
m

ite
d

Login ID:8FA03376/Student Name:ANSHULGARG/Overall Score:12

ng

Question: 77
Refer to the data given below and answer the questions.
If Sameer and Keshav are siblings then where does Krupa sit?
Third to the left of Sameer

2)

Third to the right of Rakesh

3)

Second to the right of Madhav

4)

Third to the right of Keshav

Le

ar

ni

1)

IM

Explanation:

IMS Learning Resources Pvt.Ltd.,Mumbai.All copyrights to this material vestswith IMS Learning Resources Pvt.Ltd.
No part of this materials either in part oras a whole shall be copied,printed,electronically reproduced,sold or distributed without the written
consent of IMS Learing Resources Pvt.Ltd.and any such violation would entail initiation of suitable legal proceedings.

Copyright

ou

rc

es

Pr

iv
at
e

Li
m

ite
d

Login ID:8FA03376/Student Name:ANSHULGARG/Overall Score:12

es

From fig (1) Krupa sits second to the right of Madhav. Hence, [3].

Mihir

3)

Sudhir

4)

Juhi

IM

Explanation:

ni

2)

ar

Keshav

Le

1)

ng

Question: 78
Refer to the data given below and answer the questions.
If Rakesh sits next to Sameer then who sits second to the right of Madhav?

IMS Learning Resources Pvt.Ltd.,Mumbai.All copyrights to this material vestswith IMS Learning Resources Pvt.Ltd.
No part of this materials either in part oras a whole shall be copied,printed,electronically reproduced,sold or distributed without the written
consent of IMS Learing Resources Pvt.Ltd.and any such violation would entail initiation of suitable legal proceedings.

Copyright

ou

rc

es

Pr

iv
at
e

Li
m

ite
d

Login ID:8FA03376/Student Name:ANSHULGARG/Overall Score:12

es

From fig (2) Juhi will sit second to the right of Madhav. Hence, [4].

Only I

2)

Only II

3)

Both I and III

Both I and II

IM

4)

Le

1)

ar

ni

ng

Question: 79
Refer to the data given below and answer the questions.
Which of these statements is sufficient to determine the exact seating positions of all the
eight people?
I. Rakesh sits to the immediate right of Madhav
II. Madhav sits opposite Keshav
III. Krupa sits third to the right of Sameer

Explanation:

IMS Learning Resources Pvt.Ltd.,Mumbai.All copyrights to this material vestswith IMS Learning Resources Pvt.Ltd.
No part of this materials either in part oras a whole shall be copied,printed,electronically reproduced,sold or distributed without the written
consent of IMS Learing Resources Pvt.Ltd.and any such violation would entail initiation of suitable legal proceedings.

Copyright

ou

rc

es

Pr

iv
at
e

Li
m

ite
d

Login ID:8FA03376/Student Name:ANSHULGARG/Overall Score:12

es

Only statement I gives us a fixed seating arrangement for all the eight people. Statement II and III give us multiple seating arrangements.
Hence, [1].

ng

Question: 80
Refer to the data given below and answer the questions.
Who sits to the immediate left of Sameer?
Either Mihir or Rakesh

2)

Either Juhi or Rakesh

3)

Either Juhi or Madhav or Rakesh

4)

Either Mihir or Rakesh or Juhi

Le

ar

ni

1)

IM

Explanation:

IMS Learning Resources Pvt.Ltd.,Mumbai.All copyrights to this material vestswith IMS Learning Resources Pvt.Ltd.
No part of this materials either in part oras a whole shall be copied,printed,electronically reproduced,sold or distributed without the written
consent of IMS Learing Resources Pvt.Ltd.and any such violation would entail initiation of suitable legal proceedings.

Copyright

ou

rc

es

Pr

iv
at
e

Li
m

ite
d

Login ID:8FA03376/Student Name:ANSHULGARG/Overall Score:12

es

From both the figures either Mihir or Juhi or Rakesh can sit to the immediate left of Sameer. Hence, [4].

[1]

IM

1)

Le

ar

ni

ng

Question: 81
In the question below, a statement is followed by three courses of action numbered I, II and III. A course of action is a step or
administrative decision to be taken for improvement, follow-up or further action in regard to the problem. On the basis of the information
given, decide which of the suggested courses of action logically follows.
Mark [1], if only I follows.
Mark [2], if only II follows.
Mark [3], if only III follows.
Mark [4], if II and III follow.
The decreasing sales of product X has become a cause of concern for ABC Pvt. Ltd.
I. The company should immediately discontinue product X.
II. The company should conduct a market study to understand the reason for the decrease in sales.
III. The company should reduce the price of product X by 50 percent.

2)

[2]

3)

[3]

4)

[4]

Explanation:
Only II is a possible course of action. By conducting a market study, the company can find out why the product is not doing well and
accordingly take steps to improve sales. I is too extreme. Reducing the price of the product is not advisable as it we don't know whether

IMS Learning Resources Pvt.Ltd.,Mumbai.All copyrights to this material vestswith IMS Learning Resources Pvt.Ltd.
No part of this materials either in part oras a whole shall be copied,printed,electronically reproduced,sold or distributed without the written
consent of IMS Learing Resources Pvt.Ltd.and any such violation would entail initiation of suitable legal proceedings.

Copyright

Login ID:8FA03376/Student Name:ANSHULGARG/Overall Score:12

the price of the product is a reason for the decreasing sales. Hence, [2].

2)

[2]

3)

[3]

4)

[4]

iv
at
e

[1]

Pr

1)

Li
m

ite
d

Question: 82
In the question below, a statement is followed by three courses of action numbered I, II and III. A course of action is a step or
administrative decision to be taken for improvement, follow-up or further action in regard to the problem. On the basis of the information
given, decide which of the suggested courses of action logically follows.
Mark [1], if only I follows.
Mark [2], if only II follows.
Mark [3], if only III follows.
Mark [4], if none follow.
People all over the world are outraged by reports of the US government's Internet surveillance
program that monitors their Internet usage and collects and stores their personal data like
their emails and chat transcripts.
I. People should immediately stop using the Internet.
II. People should start using code words in their emails and chats.
III. People should share only limited information on the Internet.

rc

es

Explanation:
Considering the wide range of services that the Internet provides, I is unreasonable. II is also
impractical. If someone can develop software to monitor and collect such vast data, can they
not find out the meaning of the code words? Only III is a possible and practical course of
action. Hence, [3].

[1]

2)

[2]

3)

[3]

4)

Le

1)

ar

ni

ng

es

ou

Question: 83
In the question below, a statement is followed by three courses of action numbered I, II and III. A course of action is a step or
administrative decision to be taken for improvement, follow-up or further action in regard to the problem. On the basis of the information
given, decide which of the suggested courses of action logically follows.
Mark [1], if only I follows.
Mark [2], if only II follows.
Mark [3], if only III follows.
Mark [4], if none follow.
There has been an increase in cases of thefts on long-distance trains.
I. The railway authorities should stop operating long-distance trains.
II. The railway authorities should set up a passenger complaint cell at all major halts.
III. The railway authorities should operate long-distance trains only during the day time.

IM

[4]

Explanation:
None of the given courses of action would help in reducing thefts on long-distance trains. I and III are absurd. Passenger complaint cells
generally deal with complaints regarding the services provided by the railways or its staff whereas thefts fall under the purview of the
railway police. Hence, [4].
Question: 84
In the question below, a statement is followed by three courses of action numbered I, II and III. A course of action is a step or

IMS Learning Resources Pvt.Ltd.,Mumbai.All copyrights to this material vestswith IMS Learning Resources Pvt.Ltd.
No part of this materials either in part oras a whole shall be copied,printed,electronically reproduced,sold or distributed without the written
consent of IMS Learing Resources Pvt.Ltd.and any such violation would entail initiation of suitable legal proceedings.

Copyright

Login ID:8FA03376/Student Name:ANSHULGARG/Overall Score:12

[1]

2)

[2]

3)

[3]

4)

[4]

Li
m

1)

ite
d

administrative decision to be taken for improvement, follow-up or further action in regard to the problem. On the basis of the information
given, decide which of the suggested courses of action logically follows.
Mark [1], if only I follows.
Mark [2], if only II follows.
Mark [3], if only III follows.
Mark [4], if II and III follow.
Most individuals in the corporate sector are not prepared to handle ethical issues.
I.
Individuals who are not prepared to handle ethical issues should not be allowed to enter the corporate sector.
II. Ethics should be made a compulsory subject at all levels of education.
III. Companies should lay down guidelines on how ethical issues are to be handled.

Pr

iv
at
e

Explanation:
Only II and III are possible courses of action. If people are taught
ethics at school and college they would be better prepared to handle ethical issues when they
enter the corporate sector. If there is any gap in an employee's knowledge or understanding
of ethics, it can be plugged by III. I is too extreme.
Hence, [4].

[2]

3)

[3]

4)

[4]

ar

2)

Le

[1]

1)

ni

ng

es

ou

rc

es

Question: 85
In the question below, a statement is followed by three courses of action numbered I, II and III. A course of action is a step or
administrative decision to be taken for improvement, follow-up or further action in regard to the problem. On the basis of the information
given, decide which of the suggested courses of action logically follows.
Mark [1], if only I follows.
Mark [2], if only II follows.
Mark [3], if only III follows.
Mark [4], if II and III follow.
Many people feel that governments of countries round the world are not taking environmental
issues seriously enough.
I. People should take matters into their own hands.
II. People should hold demonstrations and protests against such governments.
III. People should migrate to countries where the government takes environmental issues seriously.

IM

Explanation:
Only II is a possible and advisable course of action that will force the governments to pay attention to the issue. I is very vague and it
should also be noted that people do not have the decision making powers that governments do, so taking matters into their own hands will
not solve the main problem i.e. apathy of the governments. III is a highly impractical course of action. Hence, [2].
Question: 86
Refer to the data below and answer the questions that follow.
'a % b' means 'a is neither smaller than nor equal to b'
'a $ b' means 'a is not greater than b'

IMS Learning Resources Pvt.Ltd.,Mumbai.All copyrights to this material vestswith IMS Learning Resources Pvt.Ltd.
No part of this materials either in part oras a whole shall be copied,printed,electronically reproduced,sold or distributed without the written
consent of IMS Learing Resources Pvt.Ltd.and any such violation would entail initiation of suitable legal proceedings.

Copyright

Login ID:8FA03376/Student Name:ANSHULGARG/Overall Score:12

[1]

2)

[2]

3)

[3]

4)

[4]

Li
m

1)

ite
d

'a # b' means 'a is neither greater nor smaller than b'
'a ^ b' means 'a is not smaller than b'
'a & b' means 'a is neither greater than nor equal to b'.
Mark [1], if only conclusion I is true.
Mark [2], if only conclusion II is true.
Mark [3], if both conclusions are true.
Mark [4], if neither of the conclusions is true.
a % b, b # c, c ^ d
I. a ^ d
II. b % d

2)

[2]

3)

[3]

4)

[4]

rc

es

[1]

1)

ou

Question: 87
Refer to the data below and answer the questions that follow.
P $ Q, Q & R, S % R
I. S % P II. R ^ P

es

Pr

iv
at
e

Explanation:

Le

ar

ni

ng

Explanation:

IM

Question: 88
Refer to the data below and answer the questions that follow.
K # L, L $ M, M ^ N
I. L $ N II. K & M
1)

[1]

2)

[2]

3)

[3]

IMS Learning Resources Pvt.Ltd.,Mumbai.All copyrights to this material vestswith IMS Learning Resources Pvt.Ltd.
No part of this materials either in part oras a whole shall be copied,printed,electronically reproduced,sold or distributed without the written
consent of IMS Learing Resources Pvt.Ltd.and any such violation would entail initiation of suitable legal proceedings.

Copyright

Login ID:8FA03376/Student Name:ANSHULGARG/Overall Score:12

4)

[4]

[2]

3)

[3]

4)

[4]

iv
at
e
Pr
es
rc

2)

es

[1]

1)

ou

Question: 89
Refer to the data below and answer the questions that follow.
'a % b' means 'a is neither smaller than nor equal to b'
'a $ b' means 'a is not greater than b'
'a # b' means 'a is neither greater nor smaller than b'
'a ^ b' means 'a is not smaller than b'
'a & b' means 'a is neither greater than nor equal to b'.
Mark [1], if only conclusion I is true.
Mark [2], if only conclusion II is true.
Mark [3], if both conclusions are true.
Mark [4], if neither of the conclusions is true.
W $ X, Y % X, Z % Y
I. Z % X II. Y # W

Li
m

ite
d

Explanation:

Le

ar

ni

ng

Explanation:

IM

Question: 90
Refer to the data below and answer the questions that follow.
'a % b' means 'a is neither smaller than nor equal to b'
'a $ b' means 'a is not greater than b'
'a # b' means 'a is neither greater nor smaller than b'
'a ^ b' means 'a is not smaller than b'
'a & b' means 'a is neither greater than nor equal to b'.
Mark [1], if only conclusion I is true.
Mark [2], if only conclusion II is true.

IMS Learning Resources Pvt.Ltd.,Mumbai.All copyrights to this material vestswith IMS Learning Resources Pvt.Ltd.
No part of this materials either in part oras a whole shall be copied,printed,electronically reproduced,sold or distributed without the written
consent of IMS Learing Resources Pvt.Ltd.and any such violation would entail initiation of suitable legal proceedings.

Copyright

Login ID:8FA03376/Student Name:ANSHULGARG/Overall Score:12

Mark [3], if both conclusions are true.


Mark [4], if neither of the conclusions is true.
a # b, b ^c, c # d
I. d $ a II. a ^ c
[1]

2)

[2]

3)

[3]

4)

[4]

ite
d

1)

iv
at
e

Li
m

Explanation:

2)

[2]

3)

[3]

4)

[4]

es

[1]

1)

ou

rc

es

Pr

Question: 91
Each question has two statements (A) and (B). These two statements may be either independent causes or effects of independent causes
or a common cause. One of these statements may be the effect of the other statement. Read each statement and decide which of the
following answer choices correctly depicts the relationship between these two statements and then:
Mark [1], if statement (A) is the cause and statement (B) is its effect.
Mark [2], if statement (B) is the cause and statement (A) is its effect.
Mark [3], if both statements are independent causes.
Mark [4], if both statements are effects of independent causes.
A. Summer months are hot.
B. The earth's spin axis is tilted towards the sun in summer.

ni

ng

Explanation:
If the earth's axis is tilted towards the sun, the sun's rays would hit the earth the most at this time. So (B) is the cause and (A) is its effect.
Hence, [2].

IM

Le

ar

Question: 92
The sentences given below form a meaningful passage/flow of thought. One of the sentences is missing. Find out which sentence would
fit the missing part.
A. Thyroid disorder is the second most common endocrine gland disease in humans; thefirst being diabetes mellitus.
B. ____________________.
C. Thyroid problems are significantly higher among women; be it thyroid over-activity,under-activity or swelling.
D. This is thought to be due to the influence of sex hormones, though the exact reason isnot yet clear.
Which of the following would fit (B)?
1)

This may be due to reasons distinctive to our society.

2)

Knowing the symptoms and risk factors is half the battle.

3)

However, unlike diabetes, thyroid disorder has a distinct 'gender bias'.

4)

Fortunately, most thyroid disorders have easy and effective treatment.

IMS Learning Resources Pvt.Ltd.,Mumbai.All copyrights to this material vestswith IMS Learning Resources Pvt.Ltd.
No part of this materials either in part oras a whole shall be copied,printed,electronically reproduced,sold or distributed without the written
consent of IMS Learing Resources Pvt.Ltd.and any such violation would entail initiation of suitable legal proceedings.

Copyright

Login ID:8FA03376/Student Name:ANSHULGARG/Overall Score:12

Explanation:
he paragraph starts by talking of two disorders caused by the malefaction of the endocrine gland - thyroid disorder and diabetes mellitus.
Then C talks of how thyroid disorder is more pronounced in women and then D states a probable reason for this. Ideally, B should be
about how thyroid disorder is different from diabetes in terms of its occurrence is a particular gender group. Only [3] does that. Social
factors are not at all mentioned in the passage; so [1] is eliminated. [2] and [4] can come after D but not before it. Hence, [3].

Not many realise how eating patterns change when one is stressed.

2)

So how does one let go and avoid stress?

3)

Every individual has his/her own coping mechanism.

4)

Getting angry more often is also a sign of stress.

Pr

1)

iv
at
e

Li
m

ite
d

Question: 93
The sentences given below form a meaningful passage/flow of thought. One of the sentences is missing. Find out which sentence would
fit the missing part.
A. ________________.
B. First identify the triggers and set them right.
C. For instance, if the stress is due to work pressure, then one needs to identify whether itis a bad boss, time management issues or lack
of capability.
D. Then monitor and seek help to cope.
E. Systematic desensitisation is a technique used for phobias and anxiety.
Which of the following would fit (A)?

ou

rc

es

Explanation:
Option [1] is eliminated because it talks of eating patterns when one is stressed and is not related to the passage which is about stress in
general. B, D and E have a suggestive tone - you should do this, and then do that. It cannot be preceded by a factual statement like [3]. [4]
is also not related to the passage. With [2] as the first sentence, the passage flows smoothly: how one can avoid stress in A is answered
by the subsequent sentences. Hence, [2].

ni

ng

es

Question: 94
The sentences given below form a meaningful passage/flow of thought. One of the sentences is missing. Find out which sentence would
fit the missing part.
A. Few will argue that there is no case for reforms in the way water is managed as aresource in India.
B. In urban and semi-urban areas, the lack of adequate public investments has weakenedmunicipal systems.
C. This has led to commodification and unsustainable extraction from aquifers for risingrates of profit.
D. In this context, the proposal to separate groundwater rights from land title by amendingthe Indian Easements Act, 1882 merits serious
consideration.
E. _________________.
Which of the following would fit (E)?
Coming up with an alternative framework acceptable to all stakeholders, however, is a big challenge.

2)

As a finite, life-giving resource, access to water must remain a fundamental right.

3)

It is such a strong, rights-based approach that should underpin official policy on water in India.

4)

Private sector water services have clearly failed in many countries.

Le

ar

1)

IM

Explanation:
Sentence A indicates that the issue of water management is a major concern area in India as no reforms seem to work effectively in this
area. Then, the specific problems are stated in B and C. D talks of the government's decision to amend the Indian easements Act, 1882
and says that this step is actually is a serious one. The next sentence should provide a reason for the statement in D. [4] is beyond the
purview of the passage. [3] does not fit in because the antecedent of the pronoun 'it' is not clear. [2] is a generic statement that speaks of
fundamental rights and does not specifically allude to the plans for amendment mentioned in D. Since the government has decided to
amend the Indian easements Act, 1882 with an aim to separate groundwater rights from land title, we can say that this is an alternate to
the current act. Therefore [1], with its reference to this 'alternate framework' and 'all stakeholders', is an apt statement to follow D. Hence,
[1].

IMS Learning Resources Pvt.Ltd.,Mumbai.All copyrights to this material vestswith IMS Learning Resources Pvt.Ltd.
No part of this materials either in part oras a whole shall be copied,printed,electronically reproduced,sold or distributed without the written
consent of IMS Learing Resources Pvt.Ltd.and any such violation would entail initiation of suitable legal proceedings.

Copyright

Login ID:8FA03376/Student Name:ANSHULGARG/Overall Score:12

ite
d

Question: 95
The sentences given below form a meaningful passage/flow of thought. One of the sentences is missing. Find out which sentence would
fit the missing part.
A. ___________________.
B. Many songs were composed in different Indian languages by different writers to mobilize
people to challenge the British regime.
C. These songs reflected the country's glorious past, pointed out the oppressive
conditionsprevailing then, and urged people to free the country and shape its future.
D. Among them, one stood out to become the national anthem of independent India.
E. This year marks the centenary year of this song.
When India became independent, the newly formed country needed its own official anthem

2)

There are now guidelines to the singing and playing of the national anthem

3)

Music and songs played an important role in India's anti-colonial struggle.

4)

Choosing the national anthem became as important as that of having a national flag in 1947.

iv
at
e

Li
m

1)

Pr

Explanation:
The given excerpt, mainly sentences B and C, states the importance of songs in mobilising public opinion against the oppressive British
rule. [1] and [4] are eliminated because they chronologically do not fit in as the introductory sentence of the paragraph. [2] is about the
scenario now and is not an appropriate sentence to precede B or C. Only [3] appropriately fits in with the theme of the passage. Hence,
[3].

ng

es

ou

rc

es

Question: 96
The sentences given below form a meaningful passage/flow of thought. One of the sentences is missing. Find out which sentence would
fit the missing part.
A. Going by the state of social media affairs, one does come to realise that a fad isn't
called a fad for nothing.
B. Just like people hopped from Hi5 to Orkut to Facebook, trends change and what's here
today is gone tomorrow.
C. Maybe the death knell hasn't been sounded yet for social networking but it is
definitely losing its sheen, probably paving way for another fad.
D. _______________.
Which of the following would fit (D)?
There are those who have made a conscious decision to spend lesser time on these sites than normal.

2)

Alas! India has started experiencing social-media fatigue.

3)

It wasn't too long ago when all that people ever discussed was related to the social media.

4)

Next is what, we wonder!

Le

ar

ni

1)

IM

Explanation:
The word 'fad' means temporary fashion, especially one followed enthusiastically by a group. So, A gives an idea that the author is talking
about the dwindling popularity of the social media. Then B gives an example of how trends in social media change and C sums up the
situation and states how another trend may be on the way. There is no indication as to what would be the next popular choice. So, D
should highlight this uncertainty as well as look forward to the future. [1] and [2] reiterate the information already given in the excerpt. [3]
can begin the discussion but cannot come at the end. Only [4] fits D appropriately. Hence, [4].
Question: 97
Refer to the data below and answer the questions thatfollow.
300 countries take part in the Olympics, the Asian and the Commonwealth games. 60 countries take part in the Asian and the c
ommonwealth games. 105 countries participate in the Olympics and the Commonwealth games while 90 countries take part in the Asian

IMS Learning Resources Pvt.Ltd.,Mumbai.All copyrights to this material vestswith IMS Learning Resources Pvt.Ltd.
No part of this materials either in part oras a whole shall be copied,printed,electronically reproduced,sold or distributed without the written
consent of IMS Learing Resources Pvt.Ltd.and any such violation would entail initiation of suitable legal proceedings.

Copyright

Login ID:8FA03376/Student Name:ANSHULGARG/Overall Score:12

and the Olympics and 20 countries participate in all the three events.
How many countries participate only in Olympics?
1)

35

2)

25

3)

63

4)

Cannot be determined

Pr

iv
at
e

Li
m

ite
d

Explanation:

The question cannot be answered due to insufficient data. Hence, [4].

2)

215

3)

255

4)

85

es

225

1)

ou

rc

es

Question: 98
Refer to the data below and answer the questions thatfollow.
If every country which participates only in one of the events, wins one of each gold, silver and bronze, what is the total number of medals
won by these nations?

IM

Le

ar

ni

ng

Explanation:

Question: 99

IMS Learning Resources Pvt.Ltd.,Mumbai.All copyrights to this material vestswith IMS Learning Resources Pvt.Ltd.
No part of this materials either in part oras a whole shall be copied,printed,electronically reproduced,sold or distributed without the written
consent of IMS Learing Resources Pvt.Ltd.and any such violation would entail initiation of suitable legal proceedings.

Copyright

Login ID:8FA03376/Student Name:ANSHULGARG/Overall Score:12

I and II only

2)

I, II, and III only

3)

I, III, and IV only

4)

I amd IV only

Li
m

1)

ite
d

Refer to the data below and answer the questions that follow.
Five one rupee coins, five fifty-paise coins and five twenty-five paise coins have been randomly distributed among A, B, and C such that
all of them get equal number of coins.

Pr

iv
at
e

Explanation:

I only

2)

I, II, and III only

3)

II and III only

4)

II, III, and IV only

IM

Le

ar

ni

Explanation:

ng

1)

es

ou

rc

es

Question: 100
Refer to the data below and answer the questions that follow.

Question: 101
Refer to the data below and answer the questions that follow.

IMS Learning Resources Pvt.Ltd.,Mumbai.All copyrights to this material vestswith IMS Learning Resources Pvt.Ltd.
No part of this materials either in part oras a whole shall be copied,printed,electronically reproduced,sold or distributed without the written
consent of IMS Learing Resources Pvt.Ltd.and any such violation would entail initiation of suitable legal proceedings.

Copyright

1)

I and II only

2)

II and III only

3)

IV only

4)

II and IV only

ite
d

Login ID:8FA03376/Student Name:ANSHULGARG/Overall Score:12

iv
at
e

Li
m

Explanation:

24 cm

3)

18 cm

4)

28 cm

rc

2)

ou

20 cm

es

1)

es

Pr

Question: 102
Choose the correct alternative.
A circle with centre O has a diameter of length 30cm.The length of the perpendicular from the centre of the circle to chord AB is 9cm.Find
the length of the chord AB.

IM

Le

ar

ni

ng

Explanation:

Question: 103
Choose the correct alternative.
Find the number of four digit even numbers that can be formed using the digits 0, 1, 2, 3, and 4 if the digits can be repeated more than

IMS Learning Resources Pvt.Ltd.,Mumbai.All copyrights to this material vestswith IMS Learning Resources Pvt.Ltd.
No part of this materials either in part oras a whole shall be copied,printed,electronically reproduced,sold or distributed without the written
consent of IMS Learing Resources Pvt.Ltd.and any such violation would entail initiation of suitable legal proceedings.

Copyright

Login ID:8FA03376/Student Name:ANSHULGARG/Overall Score:12

once.
1)

54

2)

375

3)

300

4)

None of these

Li
m

ite
d

Explanation:
The thousandth place can be filled by 4 numbers 1, 2, 3 and 4 as we want only 4 digit numbers. The hundredth digit place can be filled by
all the 5 numbers as repetitions are allowed. Similarly, tenth digit place can be filled by all the 5 numbers. The unit place can be filled only
by 3 numbers 0, 2 and 4 as we want only even numbers. Therefore, the total number of 4 digits even number under the given condition is
4 5 5 3 = 300. Hence, [3].

es

ou

rc

es

Pr

iv
at
e

Question: 104
Refer to the data below and answer the questions that follow.

30

3)

25

4)

15

IM

Explanation:

ni

2)

ar

35

Le

1)

ng

Considering all cities together, what is the difference (in thousands) between the total number of credit card users of Bank Y and the total
number of credit card users of Bank X?

IMS Learning Resources Pvt.Ltd.,Mumbai.All copyrights to this material vestswith IMS Learning Resources Pvt.Ltd.
No part of this materials either in part oras a whole shall be copied,printed,electronically reproduced,sold or distributed without the written
consent of IMS Learing Resources Pvt.Ltd.and any such violation would entail initiation of suitable legal proceedings.

Copyright

32.0

2)

32.5

3)

33.0

4)

33.5

Pr

1)

iv
at
e

Question: 105
Refer to the data below and answer the questions that follow.
What is the average number (in thousands) of credit card users of Bank Z across all the six cities?

Li
m

ite
d

Login ID:8FA03376/Student Name:ANSHULGARG/Overall Score:12

ar

ni

ng

es

ou

rc

es

Explanation:

46.7%

IM

1)

Le

Question: 106
Refer to the data below and answer the questions that follow.
Total number of credit card users in cities C and E is approximately what percent age higher than the total number of credit card users in
cities A and B?

2)

47.7%

3)

48.3%

4)

50.5%

Explanation:

IMS Learning Resources Pvt.Ltd.,Mumbai.All copyrights to this material vestswith IMS Learning Resources Pvt.Ltd.
No part of this materials either in part oras a whole shall be copied,printed,electronically reproduced,sold or distributed without the written
consent of IMS Learing Resources Pvt.Ltd.and any such violation would entail initiation of suitable legal proceedings.

Copyright

Pr

iv
at
e

Li
m

ite
d

Login ID:8FA03376/Student Name:ANSHULGARG/Overall Score:12

16.4%

3)

18.1%

4)

17.4%

rc

2)

ou

15.5%

es

1)

es

Question: 107
Refer to the data below and answer the questions that follow.
For city D, the number of credit card holders of Bank X is what percentage of the total credit card holders in city D?

IM

Le

ar

ni

ng

Explanation:

Question: 108

IMS Learning Resources Pvt.Ltd.,Mumbai.All copyrights to this material vestswith IMS Learning Resources Pvt.Ltd.
No part of this materials either in part oras a whole shall be copied,printed,electronically reproduced,sold or distributed without the written
consent of IMS Learing Resources Pvt.Ltd.and any such violation would entail initiation of suitable legal proceedings.

Copyright

Login ID:8FA03376/Student Name:ANSHULGARG/Overall Score:12

Refer to the data below and answer the questions that follow.
Number of credit card users of Bank Z in city B form what percentage of the total credit card users in that city?
1)

16.67%

2)

18.67%

3)

18.75%

4)

19.25%

rc

es

Pr

iv
at
e

Li
m

ite
d

Explanation:

21.2 secs

3)

20 secs

4)

14.14 secs

2)

ng

34.64 secs

ni

1)

es

ou

Question: 109
Choose the correct alternative.
A circular track is such that it can circumscribe an equilateral triangle of side 21 m. Find the time required by a person to complete one
round around the track riding on a motorcyle with a speed of 2.2 m/s.

IM

Le

ar

Explanation:

Question: 110

IMS Learning Resources Pvt.Ltd.,Mumbai.All copyrights to this material vestswith IMS Learning Resources Pvt.Ltd.
No part of this materials either in part oras a whole shall be copied,printed,electronically reproduced,sold or distributed without the written
consent of IMS Learing Resources Pvt.Ltd.and any such violation would entail initiation of suitable legal proceedings.

Copyright

Login ID:8FA03376/Student Name:ANSHULGARG/Overall Score:12

1)

8 kmph

2)

20 kmph

3)

12 kmph

4)

10 kmph

ite
d

Choose the correct alternative.

iv
at
e

Li
m

Explanation:

es

Pr

Question: 111
Choose the correct alternative.
Shyam cheats the wholeseller by using 20% higher weights on his two pan balance during buying and cheats the consumer by using 5%
lower weights on his two pan balance while selling. He sells at the price which the wholeseller quotes. Find Shyam's percentage profit.
20%

2)

5%

3)

26.31%

4)

Cannot be determined

es

ou

rc

1)

IM

Le

ar

ni

ng

Explanation:

Question: 112

IMS Learning Resources Pvt.Ltd.,Mumbai.All copyrights to this material vestswith IMS Learning Resources Pvt.Ltd.
No part of this materials either in part oras a whole shall be copied,printed,electronically reproduced,sold or distributed without the written
consent of IMS Learing Resources Pvt.Ltd.and any such violation would entail initiation of suitable legal proceedings.

Copyright

[1]

2)

[2]

3)

[3]

4)

[4]

Li
m

1)

ite
d

Login ID:8FA03376/Student Name:ANSHULGARG/Overall Score:12

Pr

iv
at
e

Explanation:

[2]

3)

[3]

4)

[4]

ng

2)

ni

[1]

ar

1)

es

ou

rc

es

Question: 113

IM

Le

Explanation:

Question: 114

IMS Learning Resources Pvt.Ltd.,Mumbai.All copyrights to this material vestswith IMS Learning Resources Pvt.Ltd.
No part of this materials either in part oras a whole shall be copied,printed,electronically reproduced,sold or distributed without the written
consent of IMS Learing Resources Pvt.Ltd.and any such violation would entail initiation of suitable legal proceedings.

Copyright

[1]

2)

[2]

3)

[3]

4)

[4]

Li
m

1)

ite
d

Login ID:8FA03376/Student Name:ANSHULGARG/Overall Score:12

ou

rc

es

Pr

iv
at
e

Explanation:

Only I

2)

Either II or both I and III

3)

Both II and III

4)

Le

1)

ar

ni

ng

es

Question: 115
Each of the following questions is followed by three statements. Determine the statements that are sufficient to answer the questions.

IM

Only II

Explanation:

IMS Learning Resources Pvt.Ltd.,Mumbai.All copyrights to this material vestswith IMS Learning Resources Pvt.Ltd.
No part of this materials either in part oras a whole shall be copied,printed,electronically reproduced,sold or distributed without the written
consent of IMS Learing Resources Pvt.Ltd.and any such violation would entail initiation of suitable legal proceedings.

Copyright

ite
d

Login ID:8FA03376/Student Name:ANSHULGARG/Overall Score:12

Both I and II

2)

Either I and II or I and III

3)

I, II and III

4)

Both II and III

rc

es

1)

Pr

iv
at
e

Li
m

Question: 116
Each of the following questions is followed by three statements. Determine the statements that are sufficient to answer the questions.

ng

es

ou

Explanation:

IM

Le

ar

ni

Question: 117
Refer to the data below and answer the questions that follow.

For how many years is the number of students pursuing part time programme in Electrical branch more than 20% of all the part time
students in that year?
1)

IMS Learning Resources Pvt.Ltd.,Mumbai.All copyrights to this material vestswith IMS Learning Resources Pvt.Ltd.
No part of this materials either in part oras a whole shall be copied,printed,electronically reproduced,sold or distributed without the written
consent of IMS Learing Resources Pvt.Ltd.and any such violation would entail initiation of suitable legal proceedings.

Copyright

Login ID:8FA03376/Student Name:ANSHULGARG/Overall Score:12

2)

3)

4)

ou

rc

es

Pr

iv
at
e

Li
m

ite
d

Explanation:

2)

39.6%

3)

41.8%

4)

41.2%

IM

Le

Explanation:

ni

40.4%

ar

1)

ng

es

Question: 118
Refer to the data below and answer the questions that follow.
What is percentage of students pursuing part time programme in Mechanical and Civil Branch students combined over the given period as
compared to total students studying in two branches?

Question: 119
Refer to the data below and answer the questions that follow.
In which year is the ratio of students studying in full time programmes to that of those in part time programme the maximum?
1)

2007

IMS Learning Resources Pvt.Ltd.,Mumbai.All copyrights to this material vestswith IMS Learning Resources Pvt.Ltd.
No part of this materials either in part oras a whole shall be copied,printed,electronically reproduced,sold or distributed without the written
consent of IMS Learing Resources Pvt.Ltd.and any such violation would entail initiation of suitable legal proceedings.

Copyright

Login ID:8FA03376/Student Name:ANSHULGARG/Overall Score:12

2)

2008

3)

2006

4)

2005

Pr

iv
at
e

Li
m

ite
d

Explanation:

53.72%

3)

54.11%

4)

54.92%

ou

2)

es

52.22%

1)

rc

es

Question: 120
Refer to the data below and answer the questions that follow.
The number of students pursuing full time programme in Civil and Production branch over the given time period as a percentage of the full
time students in all the branches is:

Le

ar

ni

ng

Explanation:

IM

Question: 121
Refer to the data below and answer the questions that follow.
What is a difference in the total number of students pursuing full time programme and part time programme in all the branches in the given
period?
1)

470

2)

420

3)

430

4)

450

IMS Learning Resources Pvt.Ltd.,Mumbai.All copyrights to this material vestswith IMS Learning Resources Pvt.Ltd.
No part of this materials either in part oras a whole shall be copied,printed,electronically reproduced,sold or distributed without the written
consent of IMS Learing Resources Pvt.Ltd.and any such violation would entail initiation of suitable legal proceedings.

Copyright

Login ID:8FA03376/Student Name:ANSHULGARG/Overall Score:12

Explanation:
Total students pursuing part time programme = 780
Total students pursuing full time programme = 1210
Required difference = 1210- 780 = 430
Hence, [3].

24.8

2)

15.4

3)

47.2

4)

1793

Li
m

1)

ite
d

Question: 122
Choose the correct alternative.

Pr

iv
at
e

Explanation:

71

3)

85

4)

55

ou

2)

es

61

1)

rc

es

Question: 123
Choose the correct alternative.

IM

Le

ar

ni

ng

Explanation:

Question: 124
Choose the correct alternative.

IMS Learning Resources Pvt.Ltd.,Mumbai.All copyrights to this material vestswith IMS Learning Resources Pvt.Ltd.
No part of this materials either in part oras a whole shall be copied,printed,electronically reproduced,sold or distributed without the written
consent of IMS Learing Resources Pvt.Ltd.and any such violation would entail initiation of suitable legal proceedings.

Copyright

Login ID:8FA03376/Student Name:ANSHULGARG/Overall Score:12

14

2)

12

3)

17

4)

13

ite
d

1)

Li
m

Explanation:

3)

4)

Pr

2)

es

rc

1)

iv
at
e

Question: 125
Choose the correct alternative.

Le

ar

ni

ng

es

ou

Explanation:

IM

Question: 126
Choose the correct alternative.

1)

729

2)

512

3)

343

4)

None of these

Explanation:

IMS Learning Resources Pvt.Ltd.,Mumbai.All copyrights to this material vestswith IMS Learning Resources Pvt.Ltd.
No part of this materials either in part oras a whole shall be copied,printed,electronically reproduced,sold or distributed without the written
consent of IMS Learing Resources Pvt.Ltd.and any such violation would entail initiation of suitable legal proceedings.

Copyright

Login ID:8FA03376/Student Name:ANSHULGARG/Overall Score:12

(272+ 27 19 + 192) 8
= (729 + 513 + 361) 8= 12824
Hence, [4].

1593.527

2)

4058.288

3)

964.131

4)

2346.432

Li
m

1)

ite
d

Question: 127
Choose the correct alternative.

Pr

iv
at
e

Explanation:

ng

es

ou

rc

es

Question: 128
Choose the correct alternative.

ni

1)

Le

ar

2)

None of these

IM

4)

3)

Explanation:

IMS Learning Resources Pvt.Ltd.,Mumbai.All copyrights to this material vestswith IMS Learning Resources Pvt.Ltd.
No part of this materials either in part oras a whole shall be copied,printed,electronically reproduced,sold or distributed without the written
consent of IMS Learing Resources Pvt.Ltd.and any such violation would entail initiation of suitable legal proceedings.

Copyright

es

Pr

iv
at
e

Li
m

ite
d

Login ID:8FA03376/Student Name:ANSHULGARG/Overall Score:12

es

ou

rc

Question: 129
Choose the correct alternative.

2)

3)

4)

None of these

IM

Le

ar

ni

Explanation:

ng

1)

IMS Learning Resources Pvt.Ltd.,Mumbai.All copyrights to this material vestswith IMS Learning Resources Pvt.Ltd.
No part of this materials either in part oras a whole shall be copied,printed,electronically reproduced,sold or distributed without the written
consent of IMS Learing Resources Pvt.Ltd.and any such violation would entail initiation of suitable legal proceedings.

Copyright

Li
m

ite
d

Login ID:8FA03376/Student Name:ANSHULGARG/Overall Score:12

Pr

iv
at
e

Question: 130
Choose the correct alternative.

rc

es

1)

ou

2)

es

3)

4)

Le

ar

ni

ng

Explanation:

IM

Question: 131
Choose the correct alternative.

1)

2554

2)

2656

3)

2827

4)

2756

Explanation:

IMS Learning Resources Pvt.Ltd.,Mumbai.All copyrights to this material vestswith IMS Learning Resources Pvt.Ltd.
No part of this materials either in part oras a whole shall be copied,printed,electronically reproduced,sold or distributed without the written
consent of IMS Learing Resources Pvt.Ltd.and any such violation would entail initiation of suitable legal proceedings.

Copyright

ite
d

Login ID:8FA03376/Student Name:ANSHULGARG/Overall Score:12

34.7 cm

2)

27.4 cm

3)

30.5 cm

4)

24.2 cm

iv
at
e

1)

Li
m

Question: 132
Choose the correct alternative.
Find the circumradius of the triangle whose sides are 60cm,11cm amd 61cm.?

es

ou

rc

es

Pr

Explanation:

ni

ng

Question: 133
Choose the correct alternative.
.
A ten digit number is such that all its digits are distinct. If the number is added to its reverse (ten digit number) then also we get a ten digit
number. Then the number 9 cannot be:
the rightmost digit

2)

the leftmost digit

3)

the rightmost second digit

4)

Both [1] and [2]

Le

ar

1)

IM

Explanation:
9 cannot be the leftmost or rightmost digit. (As the sum of the even digit number and it's reverse will become an eleven digit number).
Hence, [4].
Question: 134
Choose the correct alternative.
Two trains of equal length travel at 40 kmph and 50 kmph respectively. The difference in the time taken by the trains to cross each other
(after they just met) while travelling in the same direction and while travelling in opposite directions is 576 seconds. Find the length of the
trains?

IMS Learning Resources Pvt.Ltd.,Mumbai.All copyrights to this material vestswith IMS Learning Resources Pvt.Ltd.
No part of this materials either in part oras a whole shall be copied,printed,electronically reproduced,sold or distributed without the written
consent of IMS Learing Resources Pvt.Ltd.and any such violation would entail initiation of suitable legal proceedings.

Copyright

Login ID:8FA03376/Student Name:ANSHULGARG/Overall Score:12

1)

1 km

2)

1.2 km

3)

0.9 km

4)

1.3 km

Le

ar

ni

ng

es

ou

rc

es

Pr

iv
at
e

Li
m

ite
d

Explanation:

IM

Question: 135
Choose the correct alternative.
Eggs cost 90 paise a dozen. Pepper costs 20 paise each. An omelette consists of 3 eggs and one-fourth of a pepper. How much will the
ingredients for 8 omelettes cost?
1)

90 p

2)

Rs.1.30

3)

Rs.1.80

4)

Rs.2.20

Explanation:

IMS Learning Resources Pvt.Ltd.,Mumbai.All copyrights to this material vestswith IMS Learning Resources Pvt.Ltd.
No part of this materials either in part oras a whole shall be copied,printed,electronically reproduced,sold or distributed without the written
consent of IMS Learing Resources Pvt.Ltd.and any such violation would entail initiation of suitable legal proceedings.

Copyright

Login ID:8FA03376/Student Name:ANSHULGARG/Overall Score:12

Li
m

Mark [1], if x

ite
d

Question: 136
Each question is followed by data in the form of two statements labelled as I and II. You must decide whether the data given in the
statements are sufficient to answer the question.

2)

3)

4)

Pr

es

1)

iv
at
e

Mark [2[, if x
y
Mark [3], if x = y
Mark [4], if the relationship between A and B cannot be determined.
I.x(x + 1) = 0
II.y2+ 2 = 3y

ng

es

ou

rc

Explanation:

Mark [1], if x

ar

ni

Question: 137
Each question is followed by data in the form of two statements labelled as I and II. You must decide whether the data given in the
statements are sufficient to answer the question.
y

IM

Le

Mark [2[, if x
y
Mark [3], if x = y
Mark [4], if the relationship between A and B cannot be determined.
x = Ratio of ages of two persons aged 35 and 49.
y = Ratio of their ages 10 years hence.
1)

2)

3)

4)

Explanation:

IMS Learning Resources Pvt.Ltd.,Mumbai.All copyrights to this material vestswith IMS Learning Resources Pvt.Ltd.
No part of this materials either in part oras a whole shall be copied,printed,electronically reproduced,sold or distributed without the written
consent of IMS Learing Resources Pvt.Ltd.and any such violation would entail initiation of suitable legal proceedings.

Copyright

ite
d

Login ID:8FA03376/Student Name:ANSHULGARG/Overall Score:12

iv
at
e

Mark [1], if x

Li
m

Question: 138
Each question is followed by data in the form of two statements labelled as I and II. You must decide whether the data given in the
statements are sufficient to answer the question.

3)

4)

es

2)

rc

ou

1)

Pr

Mark [2[, if x
y
Mark [3], if x = y
Mark [4], if the relationship between A and B cannot be determined.

IM

Le

ar

ni

ng

es

Explanation:

Question: 139
Each question is followed by data in the form of two statements labelled as I and II. You must decide whether the data given in the
statements are sufficient to answer the question.

IMS Learning Resources Pvt.Ltd.,Mumbai.All copyrights to this material vestswith IMS Learning Resources Pvt.Ltd.
No part of this materials either in part oras a whole shall be copied,printed,electronically reproduced,sold or distributed without the written
consent of IMS Learing Resources Pvt.Ltd.and any such violation would entail initiation of suitable legal proceedings.

Copyright

Login ID:8FA03376/Student Name:ANSHULGARG/Overall Score:12

Mark [1], if x

2)

3)

4)

Li
m

1)

ite
d

Mark [2[, if x
y
Mark [3], if x = y
Mark [4], if the relationship between A and B cannot be determined.

rc

es

Pr

iv
at
e

Explanation:

Mark [1], if x

es

ou

Question: 140
Each question is followed by data in the form of two statements labelled as I and II. You must decide whether the data given in the
statements are sufficient to answer the question.
y

3)

4)

IM

Explanation:

ar

2)

Le

1)

ni

ng

Mark [2[, if x
y
Mark [3], if x = y
Mark [4], if the relationship between A and B cannot be determined.
x = Largest three digit even number.
y = Largest three digit composite number.

IMS Learning Resources Pvt.Ltd.,Mumbai.All copyrights to this material vestswith IMS Learning Resources Pvt.Ltd.
No part of this materials either in part oras a whole shall be copied,printed,electronically reproduced,sold or distributed without the written
consent of IMS Learing Resources Pvt.Ltd.and any such violation would entail initiation of suitable legal proceedings.

Copyright

Login ID:8FA03376/Student Name:ANSHULGARG/Overall Score:12

2)

33.3 sq. units

3)

20.3 sq. units

4)

11.3 sq. units

Li
m

9.13 sq. units

iv
at
e

1)

ite
d

Question: 141
Choose the correct alternative.

rc

es

Pr

Explanation:

es

ou

Question: 142
Choose the correct alternative.
In triangle XYZ, points V and W are the midpoints of sides XY and XZ respectively. The area of triangle XYZ is 72 m2. Find the area of the
quadrilateral VWZY.
18 m2

2)

36 m2

3)

54 m2

4)

Cannot be determined

ni

ng

1)

IM

Le

ar

Explanation:

IMS Learning Resources Pvt.Ltd.,Mumbai.All copyrights to this material vestswith IMS Learning Resources Pvt.Ltd.
No part of this materials either in part oras a whole shall be copied,printed,electronically reproduced,sold or distributed without the written
consent of IMS Learing Resources Pvt.Ltd.and any such violation would entail initiation of suitable legal proceedings.

Copyright

es

Pr

iv
at
e

Li
m

ite
d

Login ID:8FA03376/Student Name:ANSHULGARG/Overall Score:12

ou

rc

Question: 143
Choose the correct alternative.

es

1)

ng

2)

ni

3)

ar

4)

IM

Le

Explanation:

Question: 144
Refer to the data below and answer the questions that follow.

IMS Learning Resources Pvt.Ltd.,Mumbai.All copyrights to this material vestswith IMS Learning Resources Pvt.Ltd.
No part of this materials either in part oras a whole shall be copied,printed,electronically reproduced,sold or distributed without the written
consent of IMS Learing Resources Pvt.Ltd.and any such violation would entail initiation of suitable legal proceedings.

Copyright

Login ID:8FA03376/Student Name:ANSHULGARG/Overall Score:12

ite
d

1)

2)

Li
m

3)

iv
at
e

4)

IM

Le

ar

ni

ng

es

ou

rc

es

Pr

Explanation:

Question: 145
Refer to the data below and answer the questions that follow.

1)

IMS Learning Resources Pvt.Ltd.,Mumbai.All copyrights to this material vestswith IMS Learning Resources Pvt.Ltd.
No part of this materials either in part oras a whole shall be copied,printed,electronically reproduced,sold or distributed without the written
consent of IMS Learing Resources Pvt.Ltd.and any such violation would entail initiation of suitable legal proceedings.

Copyright

Login ID:8FA03376/Student Name:ANSHULGARG/Overall Score:12

2)

3)

4)

ou

rc

es

Pr

iv
at
e

Li
m

ite
d

Explanation:

es

Question: 146
Refer to the data below and answer the questions that follow.

ng

1)

4)

IM

Explanation:

ar

Le

3)

ni

2)

IMS Learning Resources Pvt.Ltd.,Mumbai.All copyrights to this material vestswith IMS Learning Resources Pvt.Ltd.
No part of this materials either in part oras a whole shall be copied,printed,electronically reproduced,sold or distributed without the written
consent of IMS Learing Resources Pvt.Ltd.and any such violation would entail initiation of suitable legal proceedings.

Copyright

iv
at
e

Li
m

ite
d

Login ID:8FA03376/Student Name:ANSHULGARG/Overall Score:12

23 kmph

3)

20 kmph

4)

22 kmph

es

2)

rc

21 kmph

ou

1)

Pr

Question: 147
Choose the correct alternative.
Rahul travels from city A to city B at a speed of 30 kmph in one hour. Then he travels from city B to city C at a speed of 18 kmph in half an
hour. He travels from city C to city D in 45 minutes at a speed of 20 kmph and takes another 45 minutes to reach city E from city D at 16
kmph. Find Rahul's average speed in the entire journey?

IM

Le

ar

ni

ng

es

Explanation:

Question: 148
Choose the correct alternative.
Three years ago, Ramesh was twice as old as Suresh. Also, Mohan's age three years ago was thrice of the present age of Suresh. If
Mohan is 19 years elder to Ramesh then find the sum of the present ages of Ramesh, Suresh and Mohan?

IMS Learning Resources Pvt.Ltd.,Mumbai.All copyrights to this material vestswith IMS Learning Resources Pvt.Ltd.
No part of this materials either in part oras a whole shall be copied,printed,electronically reproduced,sold or distributed without the written
consent of IMS Learing Resources Pvt.Ltd.and any such violation would entail initiation of suitable legal proceedings.

Copyright

Login ID:8FA03376/Student Name:ANSHULGARG/Overall Score:12

1)

69

2)

78

3)

76

4)

None of these

iv
at
e

Li
m

ite
d

Explanation:

es

Pr

Question: 149
Choose the correct alternative.
If three distinct numbers are selected at random from the set S = {1, 2, 3, 4, 5, 6, 7, 8,}, then the probability that the product of two of the
numbers is equal to the third is:

rc

1)

ou

2)

es

3)

ng

4)

IM

Le

ar

ni

Explanation:

Question: 150
Choose the correct alternative.
A trader dealing in mangoes has an inventory of 10 cartons of mangoes (each carton containing 100 mangoes). He does a quality check
on the entire lot of mangoes with him. After checking the entire lot, he realizes that in the 1st carton, the entire lot of mangoes is of good
quality. However, in the second carton, the probability of a mango being rotten is 2%. Thereafter, with each carton, the probability of a

IMS Learning Resources Pvt.Ltd.,Mumbai.All copyrights to this material vestswith IMS Learning Resources Pvt.Ltd.
No part of this materials either in part oras a whole shall be copied,printed,electronically reproduced,sold or distributed without the written
consent of IMS Learing Resources Pvt.Ltd.and any such violation would entail initiation of suitable legal proceedings.

Copyright

Login ID:8FA03376/Student Name:ANSHULGARG/Overall Score:12

mango being rotten increases by 2% points. The probability that a mango sold by him is of good quality is:
1)

90%

2)

91%

3)

89%

4)

None of these

iv
at
e

Li
m

ite
d

Explanation:

es

Pr

Question: 151
Choose the correct alternative.
From a standard pack of 52 playing cards, four cards are drawn out and kept aside. All these four cards are found to be face cards. If two
more cards are drawn out from the remaining pack what is the probability that exactly one card is a face-card?

rc

1)

ou

2)

None of these

4)

es

3)

IM

Le

ar

ni

ng

Explanation:

Question: 152
In each of the following sentences, a part has been highlighted. Beneath each sentence, four different ways of phrasing the highlighted
part have been indicated. Choose the best alternative from among the four.
The identity cards will be available from tomorrow, and can be collected during office hours.
1)

and they were collected during office hours.

IMS Learning Resources Pvt.Ltd.,Mumbai.All copyrights to this material vestswith IMS Learning Resources Pvt.Ltd.
No part of this materials either in part oras a whole shall be copied,printed,electronically reproduced,sold or distributed without the written
consent of IMS Learing Resources Pvt.Ltd.and any such violation would entail initiation of suitable legal proceedings.

Copyright

Login ID:8FA03376/Student Name:ANSHULGARG/Overall Score:12

2)

and they can be available for collection during office hours.

3)

and these will be collected during office hours.

4)

No correction required

Explanation:
The sentence talks about a future action, which eliminates option [1]. Option [2] is ambiguous and incorrectly uses 'can' in place of 'will'.
Option [3] brings in a tone of compulsion, which is not indicated in the stem sentence. The given sentence is correct in all respects.
Hence, [4].

1)

apart from adding

2)

instead of adding

3)

close to adding

4)

No correction required

iv
at
e

Li
m

ite
d

Question: 153
In each of the following sentences, a part has been highlighted. Beneath each sentence, four different ways of phrasing the highlighted
part have been indicated. Choose the best alternative from among the four.
The company's daily newspaper was an embarrassment and far away increasing to the company's clout, it became a liability for the
promoters of the company.

es

Pr

Explanation:
The sentence comments on the fact that the company's daily newspaper did not meet the expectations. In this context, 'instead of adding'
is the most appropriate. 'Apart from adding' conveys an idea opposite to the intended idea. 'Close to adding' doesn't make any sense in
the context of the statement. Hence, [2].

ou

rc

Question: 154
In each of the following sentences, a part has been highlighted. Beneath each sentence, four different ways of phrasing the highlighted
part have been indicated. Choose the best alternative from among the four.
There were many female applicants and, with excepting, all of them were proud of their feminine identity.
with due credit

2)

exceptionally

3)

without exception

4)

No correction required

ng

es

1)

ar

ni

Explanation:
The author wants to emphasise the point that 'all' the female applicants were proud of their identity. This can be done by the use of the
phrase 'without exception'. Using 'with due credit' and 'exceptionally' makes the sentence grammatically incorrect. Hence, [3].

they had expect

IM

1)

Le

Question: 155
In each of the following sentences, a part has been highlighted. Beneath each sentence, four different ways of phrasing the highlighted
part have been indicated. Choose the best alternative from among the four.
Little did they expect that the Ivory Towers would transform their precious daughter into a left-wing rebel.

2)

they did expect

3)

did they expected

4)

No correction required

Explanation:
In this sentence type, the auxiliary verb should come before the subject. Thus, 'little did they expect' is the correct usage. Hence, no
correction is required. 'They did expect' goes against the rule stated above. 'They had expect' and 'did they expected' violate the rules of

IMS Learning Resources Pvt.Ltd.,Mumbai.All copyrights to this material vestswith IMS Learning Resources Pvt.Ltd.
No part of this materials either in part oras a whole shall be copied,printed,electronically reproduced,sold or distributed without the written
consent of IMS Learing Resources Pvt.Ltd.and any such violation would entail initiation of suitable legal proceedings.

Copyright

Login ID:8FA03376/Student Name:ANSHULGARG/Overall Score:12

tenses. Hence, [4].

1)

is at least ten times what it was

2)

are at least ten times what they were

3)

is at lest ten times what the numbers were

4)

No correction required

ite
d

Question: 156
In each of the following sentences, a part has been highlighted. Beneath each sentence, four different ways of phrasing the highlighted
part have been indicated. Choose the best alternative from among the four.
The number of people visiting the malls for shopping is at least ten times what they were only five years ago.

Li
m

Explanation:
The subject of the sentence is 'the number of people', which is singular, so both the verbs should also be in the singular form. This
eliminates option [2] as it uses the plural form of the verb. Only [1] uses the correct singular form 'is' and 'was'. Hence, [1].

[2]

3)

[3]

4)

[4]

es

2)

rc

[1]

ou

1)

Pr

iv
at
e

Question: 157
Read the sentences given below carefully. Parts of the sentences have been marked as [1], [2] and [3]. If there is an error in the sentence
as a result of any of these parts, mark the relevant part as the answer. If there are no errors, mark [4].

es

Explanation:
In part [1], the adverb 'too' is incorrectly used in place of the preposition 'to'. The word does not qualify a verb or an adjective, it rather
shows the relation between two things in the sentence: 'many of us' and 'this'. Hence, [1].

Le

ar

ni

ng

Question: 158
Read the sentences given below carefully. Parts of the sentences have been marked as [1], [2] and [3]. If there is an error in the sentence
as a result of any of these parts, mark the relevant part as the answer. If there are no errors, mark [4].

[2]

IM

2)

[1]

1)

3)

[3]

4)

[4]

Explanation:
The sentence defies the rule of parallel construction. The expression in [3] should be in sync with the rest of the expressions. It should
read 'and enjoy the stability"'. Hence, [3].

IMS Learning Resources Pvt.Ltd.,Mumbai.All copyrights to this material vestswith IMS Learning Resources Pvt.Ltd.
No part of this materials either in part oras a whole shall be copied,printed,electronically reproduced,sold or distributed without the written
consent of IMS Learing Resources Pvt.Ltd.and any such violation would entail initiation of suitable legal proceedings.

Copyright

Login ID:8FA03376/Student Name:ANSHULGARG/Overall Score:12

[1]

2)

[2]

3)

[3]

4)

[4]

Li
m

1)

ite
d

Question: 159
Read the sentences given below carefully. Parts of the sentences have been marked as [1], [2] and [3]. If there is an error in the sentence
as a result of any of these parts, mark the relevant part as the answer. If there are no errors, mark [4].

iv
at
e

Explanation:
'Principle' refers to a fundamental law or truth while a 'principal' is the chief or head of a school or college. The correct word to be used in
the context of the sentence is 'principle'. Hence, [1].

[2]

3)

[3]

4)

[4]

ou

2)

es

[1]

ng

Explanation:
There is no error in the sentence. Hence, [4].

1)

rc

es

Pr

Question: 160
Read the sentences given below carefully. Parts of the sentences have been marked as [1], [2] and [3]. If there is an error in the sentence
as a result of any of these parts, mark the relevant part as the answer. If there are no errors, mark [4].

ar

ni

Question: 161
In the following question, a statement is given in direct speech. Choose the option that best converts it into indirect speech.
My coach said, "Bravo! You have won the match."
My coach applauded me, saying I had won the match.

2)

My coach said to me I had won the match.

3)

My coach applauded me, saying I have won the match.

My coach exclaimed and said I won the match.

IM

4)

Le

1)

Explanation:
[2] is wrong because it does not reflect the actual tone of the speaker. [3] is wrong because the reporting verb is in the past tense whereas
the second part is in the present tense. [4] uses the wrong expression 'exclaimed' because 'bravo' indicates he is applauding the speaker.
Only [1] changes the sentence correctly. Hence, [1].
Question: 162
From the five words given below, select the two words that are nearly the same in meaning.

IMS Learning Resources Pvt.Ltd.,Mumbai.All copyrights to this material vestswith IMS Learning Resources Pvt.Ltd.
No part of this materials either in part oras a whole shall be copied,printed,electronically reproduced,sold or distributed without the written
consent of IMS Learing Resources Pvt.Ltd.and any such violation would entail initiation of suitable legal proceedings.

Copyright

Login ID:8FA03376/Student Name:ANSHULGARG/Overall Score:12

1)

A-C

2)

D-E

3)

A-E

4)

B-E

ite
d

Explanation:
'Scheme' and 'connive' are similar in meaning. Both mean 'to plot'. Hence, [4].

A-E

2)

C-D

3)

B-C

4)

D-E

iv
at
e

1)

Li
m

Question: 163
From the five words given below, select the two words that are nearly the same in meaning.

es

Pr

Explanation:
'Extol' means 'to praise or glorify'. 'Exalt' means the same. Hence, [2].

2)

B and C

3)

A and C

4)

Le

1)

ar

ni

ng

es

ou

rc

Question: 164
Read the following passage and answer the questions that follow.
Animal carcasses are not to be dumped into the river, washermen should not use chemicals to wash dirty clothes, the river bank is not to
be used as a toilet and no polythene bags these are among the measures villagers in Rajpura, in Mathura district of Uttar Pradesh, have
planned as part of their campaign to clean up the polluted Yamuna. With people from 73 villages in the district joining the campaign, it has
become the biggest public initiative in the region against river pollution. A whole lot of agencies have been roped in, including the
panchayati department at each block, as well as at the district headquarters. Each village will have a committee with the Pradhan as the
Chairman to supervise the salient features of the campaign. The committee will have three members two panchayat members and either
the panchayat secretary or anyone interested in water conservation.
Despite being one of the most sacred rivers in India, the Yamuna in Mathura presents a picture of total neglect.

IM

Explanation:
The passage does not mention anything about the use of public toilets. The other two choices are mentioned in the passage. So,
statements B and C are correct. Hence, [2].
Question: 165
Read the following passage and answer the questions that follow.
Which of the following is the most appropriate synonym of the word 'carcass'?

IMS Learning Resources Pvt.Ltd.,Mumbai.All copyrights to this material vestswith IMS Learning Resources Pvt.Ltd.
No part of this materials either in part oras a whole shall be copied,printed,electronically reproduced,sold or distributed without the written
consent of IMS Learing Resources Pvt.Ltd.and any such violation would entail initiation of suitable legal proceedings.

Copyright

Login ID:8FA03376/Student Name:ANSHULGARG/Overall Score:12

1)

Limbs

2)

Excreta

3)

Dead body

4)

Waste

Explanation:
Carcass means 'dead body'. Hence, [3].

Sociologist

2)

Environmentalist

3)

Politician

4)

Newspaper reporter

iv
at
e

Li
m

1)

ite
d

Question: 166
Read the following passage and answer the questions that follow.
It can be inferred from the passage that the author is most likely to be a/an:

rc

es

Question: 167
Read the following passage and answer the questions that follow.
Which of the following is not true about the campaign to clean the Yamuna?

Pr

Explanation:
We can see that the passage reports some major developments in a district which aims at reducing the pollution of the river Yamuna.
There is no mention of the author being involved in any such activity. The author merely reports an episode that has taken place
somewhere. So, we can infer that the author is a 'newspaper reporter'. Hence, [4].

73 villages have joined the campaign to clean the Yamuna.

2)

Several measures have been planned by the villagers to clean the Yamuna.

3)

Laws for penalising people who dump polythene bags in the Yamuna have been framed.

4)

The panchayati department has been involved at the block level.

es

ou

1)

ng

Explanation:
The passage does not mention any law to penalise people. It only mentions that no polythene bags should be used around the Yamuna
river. Hence, this is not true about the campaign. The other choices are mentioned in the passage and they are correct. Hence, [3].

2)

Orthodox

3)

Ancient

4)

ar

Profane

Le

1)

ni

Question: 168
Read the following passage and answer the questions that follow.
Which of the following is the most appropriate antonym of the word 'sacred'?

IM

Pure

Explanation:
'Sacred' means holy or sanctified. Thus, the correct antonym of the word is 'profane', which means 'unholy' or 'irreligious'. Hence, [1].
Question: 169
From the five words given below, select the two words that are nearly opposite in meaning.

IMS Learning Resources Pvt.Ltd.,Mumbai.All copyrights to this material vestswith IMS Learning Resources Pvt.Ltd.
No part of this materials either in part oras a whole shall be copied,printed,electronically reproduced,sold or distributed without the written
consent of IMS Learing Resources Pvt.Ltd.and any such violation would entail initiation of suitable legal proceedings.

Copyright

Login ID:8FA03376/Student Name:ANSHULGARG/Overall Score:12

1)

AB

2)

CE

3)

AC

4)

BD

2)

CD

3)

AE

4)

BC

Pr

AC

es

1)

iv
at
e

Question: 170
From the five words given below, select the two words that are nearly opposite in meaning.

Li
m

ite
d

Explanation:
'Beautify' means 'to make or become beautiful'. 'Bilk means' 'to obtain or withhold money from someone unfairly or by deceit; cheat or
defraud'. 'Squabble' means 'to engage in disagreeable arguments over trivial issues'. 'Bicker' means 'to engage in petty, bad tempered
quarrel'. 'Concur' means 'agree'. So, 'sqaubble' and 'concur' and 'bicker' and 'concur' are antonyms. Hence, C and E form a pair. Hence,
[2].

ou

rc

Explanation:
'Coddle' means 'to treat with great care or to pamper'. 'Collide' means 'to strike one another with forceful impact'. 'Cosset' means 'to treat
with tenderness and care'. 'Conspire' means 'to plan secretly'. 'Harass' means 'pester' or 'torment'. It is antonymous to 'coddle' and
'cosset'. Hence, A and E form a pair. Hence, [3].

CD

3)

CA

4)

BD

ng

2)

ni

DE

ar

1)

es

Question: 171
From the five words given below, select the two words that are nearly opposite in meaning.

Le

Explanation:
'Banal' means 'commonplace or devoid of originality'. 'Trite' means 'lacking freshness or stale'. 'Taut' means 'tightly drawn or in good
condition'. 'Astute' means 'discerning or shrewd'. 'Obtuse' means 'not sharp or observant'. Thus, D and E form a pair. Hence, [1].

IM

Question: 172
In each of the following sentences, a part of the sentence is underlined. Beneath each sentence, four different ways of phrasing the
underlined part are indicated. Choose the best alternative from among the four.

1)

further discussion of the launch strategy for the new product until their next meeting.

2)

further discussion of the launch strategy for the new product until it has their next meeting.

IMS Learning Resources Pvt.Ltd.,Mumbai.All copyrights to this material vestswith IMS Learning Resources Pvt.Ltd.
No part of this materials either in part oras a whole shall be copied,printed,electronically reproduced,sold or distributed without the written
consent of IMS Learing Resources Pvt.Ltd.and any such violation would entail initiation of suitable legal proceedings.

Copyright

Login ID:8FA03376/Student Name:ANSHULGARG/Overall Score:12

3)

further discussion of the launch strategy for the new product until its next meeting.

4)

farther discussion of the launch strategy for the new product until its next meeting.

Explanation:
The given sentence has a pronoun-antecedent agreement error. The pronoun 'their' (which is plural) does not agree with the noun 'board'
(which is singular). This error is corrected only in options [3] and [4]. 'Farther', which means 'beyond', is logically and contextually
incorrect. Hence, [3].

Li
m

ite
d

Question: 173
In each of the following sentences, a part of the sentence is underlined. Beneath each sentence, four different ways of phrasing the
underlined part are indicated. Choose the best alternative from among the four.

Initially it was a protest against academic painting and then the Fauvism movement wielded great influence on the artists of
its time.

2)

Initially a protest against academic painting, great influence was wielded by the Fauvism movement on the artists of its
time.

3)

The Fauvism movement wielded great influence on the artists of its time though it was initially a protest against academic
painting.

4)

Initially a protest against academic painting, the Fauvism movement wielded great influence on the artists of its time.

Pr

iv
at
e

1)

rc

es

Explanation:
The only problem with the underlined sentence is the missing article. [4] solves this problem and is thus the right answer. [1] is
unnecessarily wordy, [2] contains a modifier error (the phrase before the comma is supposed to modify "the Fauvism movement" not
"great influence") and [3] has an unwarranted "though". Hence, [4].

es

ou

Question: 174
In each of the following sentences, a part of the sentence is underlined. Beneath each sentence, four different ways of phrasing the
underlined part are indicated. Choose the best alternative from among the four.

qualities of academic programs was generally good but a bit outdated.

2)

quality of academic programs are generally good but a bit outdated.

3)

quality of academic programs were generally good but a bit outdated.

4)

quality of academic programs was generally good but a bit outdated.

ar

ni

ng

1)

Le

Explanation:
The given sentence has a subject-verb agreement error. The verb 'were' (which is plural) does not agree with the noun 'quality' (which is
singular). This error is corrected in option [4], which also uses the appropriate conjunction 'but' to show contrast. Hence, [4].

IM

Question: 175
In each of the following sentences, a part of the sentence is underlined. Beneath each sentence, four different ways of phrasing the
underlined part are indicated. Choose the best alternative from among the four.

1)

to be experiencing

2)

to have been experiencing

3)

to experience

IMS Learning Resources Pvt.Ltd.,Mumbai.All copyrights to this material vestswith IMS Learning Resources Pvt.Ltd.
No part of this materials either in part oras a whole shall be copied,printed,electronically reproduced,sold or distributed without the written
consent of IMS Learing Resources Pvt.Ltd.and any such violation would entail initiation of suitable legal proceedings.

Copyright

Login ID:8FA03376/Student Name:ANSHULGARG/Overall Score:12

4)

to have experienced

Explanation:
There is no error in the given sentence. 'To experience' may seem correct but on closer inspection of the sentence we see that we are
talking about 'well known personalities', who would be well past their childhoods. Hence, [4].

wouldn't it?

2)

wouldn't we?

3)

would it?

4)

would we?

Li
m

1)

ite
d

Question: 176
In each of the following sentences, a part of the sentence is underlined. Beneath each sentence, four different ways of phrasing the
underlined part are indicated. Choose the best alternative from among the four.
If we knew what it was we were doing, it would not be called research, wouldn't it?

Pr

iv
at
e

Explanation:
By rule, a positive statement is followed by a negative question tag and a negative statement is followed by a positive question tag.
Question tags are formed with the auxiliary verb used in the sentence and the appropriate subject. The given statement is a negative
statement, so it should be followed by a positive tag. Between [3] and [4], [3] is the correct choice as 'it' is the subject being spoken about.
Hence, [3].

1)

He is able to continuously perform better and better.

2)

Le

ar

ni

ng

es

ou

rc

es

Question: 177
Refer to the passage given below and answer the questions that follow.
A 'choke', in sport, is a sudden occurrence of severe anxiety at a critical stage of a competition. It is typically experienced by an athlete
who is approaching victory but becomes tense and apprehensive at the prospect and loses form. The term is obviously taken from the
original sense of the word 'choke' which describes temporary or permanent suffocation, loss of breathing or paralysis due to blockage of
the throat. In sports terms, a choke is rarely temporary: the competitor suffers an acute attack and seldom has chance to recover
composure before his or her rival capitalizes.
Historically, the event that became known as The Choke involved Jana Novotna, who led Steffi Graf 4-1 in games and 40-0 with serve in
the third set of the 1993 Wimbledon women's singles final. Novotna, who had looked confidently in control of the game, crumbled and lost.
This happened time and again, in later matches. These repeated incidents raised a question: whether there are certain types of
personality that dispose the person having them to become temporarily inept because of emotion, or block up in certain types of
situations.
Sometimes, when winning becomes an active possibility, arousal levels may increase to the point where physiological functions interfere
with performance. Though all athletes are told to guard against thinking too far ahead, with victory in sight there may be a tendency to
imagine the prospect of winning. This tendency may, by itself, prompt an increase in metabolic rate, respiratory volume, a constriction of
blood vessels and perhaps heavy perspiration. Once these exceed optimum levels for athletic performance, they can lead to sharp
decrements. As form deteriorates, the relaxation that would allow them to return to optimum performance becomes difficult if not
impossible to attain.
What happens to an athlete who experiences a 'choke'?

IM

He loses concentration and focus, and is unable to take in his surroundings.

3)

He becomes tense and anxious and his performance progressively declines.

4)

He becomes overwhelmed with the situation and begins to palpitate.

Explanation:

IMS Learning Resources Pvt.Ltd.,Mumbai.All copyrights to this material vestswith IMS Learning Resources Pvt.Ltd.
No part of this materials either in part oras a whole shall be copied,printed,electronically reproduced,sold or distributed without the written
consent of IMS Learing Resources Pvt.Ltd.and any such violation would entail initiation of suitable legal proceedings.

Copyright

Login ID:8FA03376/Student Name:ANSHULGARG/Overall Score:12

ite
d

Question: 178
Refer to the passage given below and answer the questions that follow.
From where has the word 'choke' come into sport?
From the Jana Novotna-Steffi Graf incident in the 1993 Wimbledon women's singles final.

2)

It is a term unique to sport and created in sport itself.

3)

It has come form the English word 'choke' which means temporary or permanent suf-focation, loss of breath or paralysis
due to blockage of the throat.

4)

It has entered the arena of sport from medical research on personality types that are more prone to emotional disturbance.

Li
m

1)

es

Question: 179
Refer to the passage given below and answer the questions that follow.
When did the term 'choke' first come into use in sport?

Pr

iv
at
e

Explanation:
One has to first understand this question properly. The intention of the question is to ask us 'from where' (and not 'why') the word 'choke'
entered sport. It is mentioned in the passage that it comes from the English word "choke" which means suffocation, loss of breath, etc.
Hence, [3].

When Jana Novotna panicked and lost the 1993 Wimbledon men's singles final after being in a winning position.

2)

When Steffi Graf panicked and lost the 1993 Wimbledon women's singles final after being in a winning position.

3)

When Jana Novotna panicked and lost the 1993 Wimbledon women's singles semifinal after being in a winning position.

4)

When Jana Novotna panicked and lost the 1993 Wimbledon women's singles final after being in a winning position.

es

ou

rc

1)

ng

Explanation:
This question asks us 'when' the term 'choke' entered sport. Note that there are very subtle differences in each option given. But it is
mentioned in the passage that the word "choke" came into use for the first time when 'Jana Novotna panicked and lost the 1993
Wimbledon women's singles final after being in a winning position'. Hence, [4].

The athlete being in a winning position or within sight of a victory

2)

The athlete having an aggressive and confident personality

3)

The presence of overwhelming odds, overcoming which seems only a dream

4)

Le

1)

ar

ni

Question: 180
Refer to the passage given below and answer the questions that follow.
Which of the following is absolutely essential for an athlete to be said to have 'choked'?

IM

A failure on the part of the coach or captain to advise the athlete to relax in tense situations

Explanation:
The definition of the word 'choke', as it is defined in sport, is clearly indicated in the opening lines of the passage. Being in sight of victory
or approaching victory is the defining condition necessary for a 'choke'. Hence, [1].
Question: 181
Refer to the passage given below and answer the questions that follow.
Which of the following is opposite in meaning to the word 'decrement', as used in the passage?
1)

decrease

IMS Learning Resources Pvt.Ltd.,Mumbai.All copyrights to this material vestswith IMS Learning Resources Pvt.Ltd.
No part of this materials either in part oras a whole shall be copied,printed,electronically reproduced,sold or distributed without the written
consent of IMS Learing Resources Pvt.Ltd.and any such violation would entail initiation of suitable legal proceedings.

Copyright

Login ID:8FA03376/Student Name:ANSHULGARG/Overall Score:12

2)

expenditure

3)

abatement

4)

augmentation

Explanation:
'Decrement' means 'a gradual decrease'. Options [1], [2] and [3] are synonyms of the word. Only [4], which means 'an increase' is
opposite in meaning. Hence, [4].

visual

2)

cerebral

3)

psychic

4)

pedagogic

iv
at
e

1)

Li
m

ite
d

Question: 182
Fill in the missing links in the paragraph given.
Catherine's ___(31)___ abilities were increasing and she was becoming even more intuitive. She still had arguments with Stuart, but she
felt able to ___(32)___ with him more effectively. Her eyes sparkled; her skin glowed. She had a strange dream ___(33)___ the week, she
announced, but she could only remember a ___(34)___ of it. She had dreamt that the red fin of a fish was ___(35)___ in her hand.

Pr

Explanation:
What is refered to here is 'intuition'. Among the options, only 'psychic' is closest in meaning. Hence, [3].

repel

3)

reason

4)

heal

rc

2)

ou

bond

es

1)

es

Question: 183
Fill in the missing links in the paragraph given.

ng

Question: 184
Fill in the missing links in the paragraph given.

Explanation:
'Reason' is contextually apt as it alludes to the 'arguments' mentioned in the sentence. Hence, [3].

over

2)

by

3)

through

4)

during

Le

ar

ni

1)

IM

Explanation:
'During' implies 'at some time in the week' and is the appropriate choice.
Hence, [4].
Question: 185
Fill in the missing links in the paragraph given.
1)

mite

2)

fragment

3)

notion

IMS Learning Resources Pvt.Ltd.,Mumbai.All copyrights to this material vestswith IMS Learning Resources Pvt.Ltd.
No part of this materials either in part oras a whole shall be copied,printed,electronically reproduced,sold or distributed without the written
consent of IMS Learing Resources Pvt.Ltd.and any such violation would entail initiation of suitable legal proceedings.

Copyright

Login ID:8FA03376/Student Name:ANSHULGARG/Overall Score:12

4)

feel

Explanation:
Only 'fragment' makes sense because she could only remember 'a small part' of her dreams. Hence, [2].

1)

stranded

2)

rounded

3)

pierced

4)

embedded

ite
d

Question: 186
Fill in the missing links in the paragraph given.

Li
m

Explanation:
To be 'embedded' is to stay stuck inside and it is appropriate here. Hence, [4].

atypical

3)

natural

4)

common

es

2)

rc

detrimental

ou

1)

Pr

iv
at
e

Question: 187
Fill in the missing links in the paragraph given.
Repressive and persecuting passions are very ___(36)___, as the present state of the world too amply proves. But they are not an
___(37)___ part of human nature. On the contrary, they are always the ___(38)___ of some kind of unhappiness. It should be one of the
functions of a teacher to open vistas before his pupils showing them the ___(39)___ of activities that will be as delightful as they are
useful, thereby letting loose their ___(40)___ impulses and preventing the growth of a desire to rob others of joys that they will have
missed.

2)

unobjectionable

3)

inevitable

4)

offensive

ni

accepted

Le

ar

1)

ng

Question: 188
Fill in the missing links in the paragraph given.

es

Explanation:
Options [1] and [2] are irrelevant. Options [3] and [4] are close. But [4] is the correct option as [3] contradicts the next sentence. Hence,
[4].

Explanation:
Option [3] best fits the blank as 'inevitable' means 'invariably occuring or impossible to avoid'. It fits the logic best. Hence, [3].

IM

Question: 189
Fill in the missing links in the paragraph given.
1)

cause

2)

reason

3)

outcome

4)

feeling

Explanation:

IMS Learning Resources Pvt.Ltd.,Mumbai.All copyrights to this material vestswith IMS Learning Resources Pvt.Ltd.
No part of this materials either in part oras a whole shall be copied,printed,electronically reproduced,sold or distributed without the written
consent of IMS Learing Resources Pvt.Ltd.and any such violation would entail initiation of suitable legal proceedings.

Copyright

Login ID:8FA03376/Student Name:ANSHULGARG/Overall Score:12

We are talking of 'repressive and persecuting passions'. Option [3] best fits the blank as 'unhappiness' is the cause of the above. Hence,
[3].

1)

possibility

2)

necessity

3)

pertinence

4)

rightness

ite
d

Question: 190
Fill in the missing links in the paragraph given.

Li
m

Explanation:
Option [1] best fits the blank, whereas the other options are out of
context. Hence, [1].

malicious

2)

visceral

3)

kind

4)

genuine

Pr

1)

iv
at
e

Question: 191
Fill in the missing links in the paragraph given.

es

Explanation:
Option [1] is irrelevant. Option [2] does not convey the underlying meaning. Option [4] cannot be ascertained. Hence, [3].

musing

3)

quirk

4)

impulse

ou

2)

es

Caprice

1)

rc

Question: 192
Choose the odd one out.

ni

ng

Explanation:
'Caprice', 'quirk' and 'impulse' all imply 'a sudden unpredictable action'. 'Musing' which implies 'reflection' is the odd one out. Hence, [2].

1)

Modest

2)

decent

3)

Le

ar

Question: 193
Choose the correct alternative.
Shoddy is to Clean, as Resolute is to ____________.

IM

wavering

4)

strong

Explanation:
'Shoddy' means 'untidy' and its opposite is 'clean'. 'Resolute' means 'determined' and its opposite is 'wavering', i.e., shaky. Hence, [3].
Question: 194
In each of these questions, jumbled alphabets of a meaningful word are given. You are to rearrange the alphabets and select, from the
given alternatives, the word that is almost opposite in meaning to the re-arranged word.

IMS Learning Resources Pvt.Ltd.,Mumbai.All copyrights to this material vestswith IMS Learning Resources Pvt.Ltd.
No part of this materials either in part oras a whole shall be copied,printed,electronically reproduced,sold or distributed without the written
consent of IMS Learing Resources Pvt.Ltd.and any such violation would entail initiation of suitable legal proceedings.

Copyright

Login ID:8FA03376/Student Name:ANSHULGARG/Overall Score:12

IDRFLAOBEM
1)

intimidating

2)

insular

3)

benign

4)

sagacious

ite
d

Explanation:
The word formed is FORMIDABLE, which means 'terrifying or dangerous'. 'Benign', which means 'goodhearted or amiable', is its
appropriate antonym. Hence, [3].

accredit

2)

sanctify

3)

pester

4)

retract

iv
at
e

1)

Li
m

Question: 195
In each of these questions, jumbled alphabets of a meaningful word are given. You are to rearrange the alphabets and select, from the
given alternatives, the word that is almost opposite in meaning to the re-arranged word.
ARIDNO

es

Pr

Explanation:
The word formed is: ORDAIN, which means 'to establish by law'. Its opposite would be 'retract', which means 'to take back or cancel'.
Hence, [4].

es

ou

rc

Question: 196
Choose the most suitable conjunction or preposition to combine the two sentence fragments given in each question. The result should be
a single coherent sentence. Some punctuation marks have been ignored.
A] researchers showed that during sleep, cerebral spinal fluid is pumped around the brain
B] a series of experiments on mice
after

2)

through

3)

from

4)

notwithstanding

ng

1)

Le

ar

ni

Explanation:
The correct sentence can only be 'Through a series of experiments on mice, researchers showed that during sleep, cerebral spinal fluid is
pumped around the brain.' Here 'through' means 'using/with the help of'.
Hence, [2].

IM

Question: 197
Choose the most suitable conjunction or preposition to combine the two sentence fragments given in each question. The result should be
a single coherent sentence. Some punctuation marks have been ignored.
A] and yet the title, Stuff Happens, is not inappropriate
B] the narrative of the book is episodic, with a selection of interesting stuff happening in esoteric places
1)

besides

2)

although

3)

since

4)

so

IMS Learning Resources Pvt.Ltd.,Mumbai.All copyrights to this material vestswith IMS Learning Resources Pvt.Ltd.
No part of this materials either in part oras a whole shall be copied,printed,electronically reproduced,sold or distributed without the written
consent of IMS Learing Resources Pvt.Ltd.and any such violation would entail initiation of suitable legal proceedings.

Copyright

Login ID:8FA03376/Student Name:ANSHULGARG/Overall Score:12

Explanation:
The correct sentence can only be 'And yet the title, Stuff Happens, is not inappropriate since the narrative of the book is episodic, with a
selection of interesting stuff happening in esoteric places.' Fragment B gives information that makes the title i.e. Stuff Happens, seem
appropriate. So fragments A and B are not contradictory. Hence, [3].

but

2)

as

3)

for

4)

so

Li
m

1)

ite
d

Question: 198
Choose the most suitable conjunction or preposition to combine the two sentence fragments given in each question. The result should be
a single coherent sentence. Some punctuation marks have been ignored.
A] to be young was very heaven
B] bliss was it in that dawn to be alive

iv
at
e

Explanation:
The correct sentence can only be 'Bliss was it in that dawn to be alive but to be young was very heaven.' Fragment A is more intense
than B (indicated by 'very heaven'): to be alive was blissful but to be young was absolutely heavenly. Hence, [1].

es

Pr

Question: 199
Choose the most suitable conjunction or preposition to combine the two sentence fragments given in each question. The result should be
a single coherent sentence. Some punctuation marks have been ignored.

but

2)

while

3)

yet

4)

and

es

1)

ou

rc

A] earlier, we had only 35 per cent women taking part


B] people were sceptical whether women would be able to run credit groups or community organisations

ni

ng

Explanation:
The correct sentence can only be 'Earlier, we had only 35 per cent women taking part and people were sceptical whether women would
be able to run credit groups or community organisations.' Fragments A and B do not contradict each other - rather they both seem to be
effects of a common cause. Hence, [4].

provided

IM

1)

Le

ar

Question: 200
Choose the most suitable conjunction or preposition to combine the two sentence fragments given in each question. The result should be
a single coherent sentence. Some punctuation marks have been ignored.
A] the economic and security fallout from the Snowden spy scandal has yet to crystallise fully
B] there is little doubt that America's and Britain's soft power, their ability to build alliances on the claim of moral leadership, for example,
has suffered a tangible blow

2)

because

3)

while

4)

despite

Explanation:
The correct sentence can only be 'While the economic and security fallout from the Snowden spy scandal has yet to crystallise fully, there
is little doubt that America's and Britain's soft power, their ability to build alliances on the claim of moral leadership for example, has

IMS Learning Resources Pvt.Ltd.,Mumbai.All copyrights to this material vestswith IMS Learning Resources Pvt.Ltd.
No part of this materials either in part oras a whole shall be copied,printed,electronically reproduced,sold or distributed without the written
consent of IMS Learing Resources Pvt.Ltd.and any such violation would entail initiation of suitable legal proceedings.

Copyright

Login ID:8FA03376/Student Name:ANSHULGARG/Overall Score:12

IM

Le

ar

ni

ng

es

ou

rc

es

Pr

iv
at
e

Li
m

ite
d

suffered a tangible blow.' Here 'while' means 'even though'. 'Despite the fact that' could have also been used, but not just 'despite' on its
own. Hence, [3].

IMS Learning Resources Pvt.Ltd.,Mumbai.All copyrights to this material vestswith IMS Learning Resources Pvt.Ltd.
No part of this materials either in part oras a whole shall be copied,printed,electronically reproduced,sold or distributed without the written
consent of IMS Learing Resources Pvt.Ltd.and any such violation would entail initiation of suitable legal proceedings.

Copyright

Anda mungkin juga menyukai